Правила уравнений умножения и деления. Линейные уравнения с двумя переменными

Уравнения

Как решать уравнения?

В этом разделе мы вспомним (или изучим – уж кому как) самые элементарные уравнения. Итак, что такое уравнение? Говоря человеческим языком, это какое-то математическое выражение, где есть знак равенства и неизвестное. Которое, обычно, обозначается буквой «х» . Решить уравнение - это найти такие значения икса, которые при подстановке в исходное выражение, дадут нам верное тождество. Напомню, что тождество – это выражение, которое не вызывает сомнения даже у человека, абсолютно не отягощенного математическими знаниями. Типа 2=2, 0=0, ab=ab и т.д. Так как решать уравнения? Давайте разберёмся.

Уравнения бывают всякие (вот удивил, да?). Но всё их бесконечное многообразие можно разбить всего на четыре типа.

4. Все остальные.)

Всех остальных, разумеется, больше всего, да...) Сюда входят и кубические, и показательные, и логарифмические, и тригонометрические и всякие другие. С ними мы в соответствующих разделах плотно поработаем.

Сразу скажу, что иногда и уравнения первых трёх типов так накрутят, что и не узнаешь их… Ничего. Мы научимся их разматывать.

И зачем нам эти четыре типа? А затем, что линейные уравнения решаются одним способом, квадратные другим, дробные рациональные - третьим, а остальные не решаются вовсе! Ну, не то, чтобы уж совсем никак не решаются, это я зря математику обидел.) Просто для них существуют свои специальные приёмы и методы.

Но для любых (повторяю - для любых! ) уравнений есть надёжная и безотказная основа для решения. Работает везде и всегда. Эта основа - Звучит страшно, но штука очень простая. И очень (очень!) важная.

Собственно, решение уравнения и состоит из этих самых преобразований. На 99%. Ответ на вопрос: "Как решать уравнения? " лежит, как раз, в этих преобразованиях. Намёк понятен?)

Тождественные преобразования уравнений.

В любых уравнениях для нахождения неизвестного надо преобразовать и упростить исходный пример. Причем так, чтобы при смене внешнего вида суть уравнения не менялась. Такие преобразования называются тождественными или равносильными.

Отмечу, что эти преобразования относятся именно к уравнениям. В математике ещё имеются тождественные преобразования выражений. Это другая тема.

Сейчас мы с вами повторим все-все-все базовые тождественные преобразования уравнений.

Базовые потому, что их можно применять к любым уравнениям – линейным, квадратным, дробным, тригонометрическим, показательным, логарифмическим и т.д. и т.п.

Первое тождественное преобразование: к обеим частям любого уравнения можно прибавить (отнять) любое (но одно и то же!) число или выражение (в том числе и выражение с неизвестным!). Суть уравнения от этого не меняется.

Вы, между прочим, постоянно пользовались этим преобразованием, только думали, что переносите какие-то слагаемые из одной части уравнения в другую со сменой знака. Типа:

Дело знакомое, переносим двойку вправо, и получаем:

На самом деле вы отняли от обеих частей уравнения двойку. Результат получается тот же самый:

х+2 - 2 = 3 - 2

Перенос слагаемых влево-вправо со сменой знака есть просто сокращённый вариант первого тождественного преобразования. И зачем нам такие глубокие познания? – спросите вы. В уравнениях низачем. Переносите, ради бога. Только знак не забывайте менять. А вот в неравенствах привычка к переносу может и в тупик поставить….

Второе тождественное преобразование : обе части уравнения можно умножить (разделить) на одно и то же отличное от нуля число или выражение. Здесь уже появляется понятное ограничение: на ноль умножать глупо, а делить и вовсе нельзя. Это преобразование вы используете, когда решаете что-нибудь крутое, типа

Понятное дело, х = 2. А вот как вы его нашли? Подбором? Или просто озарило? Чтобы не подбирать и не ждать озарения, нужно понять, что вы просто поделили обе части уравнения на 5. При делении левой части (5х) пятёрка сократилась, остался чистый икс. Чего нам и требовалось. А при делении правой части (10) на пять, получилась, знамо дело, двойка.

Вот и всё.

Забавно, но эти два (всего два!) тождественных преобразования лежат в основе решения всех уравнений математики. Во как! Имеет смысл посмотреть на примерах, что и как, правда?)

Примеры тождественных преобразований уравнений. Основные проблемы.

Начнём с первого тождественного преобразования. Перенос влево-вправо.

Пример для младшеньких.)

Допустим, надо решить вот такое уравнение:

3-2х=5-3х

Вспоминаем заклинание: "с иксами - влево, без иксов - вправо!" Это заклинание - инструкция по применению первого тождественного преобразования.) Какое выражение с иксом у нас справа? ? Ответ неверный! Справа у нас - ! Минус три икс! Стало быть, при переносе влево, знак поменяется на плюс. Получится:

3-2х+3х=5

Так, иксы собрали в кучку. Займёмся числами. Слева стоит тройка. С каким знаком? Ответ "с никаким" не принимается!) Перед тройкой, действительно, ничего не нарисовано. А это значит, что перед тройкой стоит плюс. Так уж математики договорились. Ничего не написано, значит, плюс. Следовательно, в правую часть тройка перенесётся с минусом. Получим:

-2х+3х=5-3

Остались сущие пустяки. Слева - привести подобные, справа - посчитать. Сразу получается ответ:

В этом примере хватило одного тождественного преобразования. Второе не понадобилось. Ну и ладно.)

Пример для старшеньких.)

Если Вам нравится этот сайт...

Кстати, у меня есть ещё парочка интересных сайтов для Вас.)

Можно потренироваться в решении примеров и узнать свой уровень. Тестирование с мгновенной проверкой. Учимся - с интересом!)

можно познакомиться с функциями и производными.

Уравнение - это равенство, содержащее букву, знамение которой нужно найти. Решение уравнения - это тот набор значений букв, при котором уравнение превращается в верное равенство:

Напомним, что для решения уравнении надо слагаемые с неизвестным перенести в одну часть равенства, а числовые слагаемые в другую, привести подобные и получить такое равенство:

Из последнего равенства определим неизвестное по правилу: «один из множителей равен частному, деленному на второй множитель».

Так как рациональные числа а и Ь могут иметь одинаковые и разные знаки, то знак неизвестного определяется по правилам деления рациональных чисел.

Порядок решения линейных уравнений

Линейное уравнение необходимо упростить, раскрыв скобки и выполнив действия второй ступени (умножение и деление).

Перенести неизвестные в одну сторону от знака равенства, а числа - в другую сторону от знака равенства, получив тождественное заданному равенство,

Привести подобные слева и справа от знака равенства, получив равенство вида ax = b .

Вычислить корень уравнения (найти неизвестное х из равенства x = b : a ),

Выполнить проверку, подставив неизвестное в заданное уравнение.

Если получим тождество в числовом равенстве, то уравнение решено верно.

Особые случаи решения уравнений

  1. Если уравнение задано произведением, равным 0, то для его решения используем свойство умножения: «произведение равно нулю, если один из сомножителей или оба сомножителя равны нулю».

27 (x - 3) = 0
27 не равно 0, значит x - 3 = 0

У второго примера два решения уравнения, так как
это уравнение второй степени:

Если коэффициенты уравнения являются обыкновенными дробями, то прежде всего надо избавиться от знаменателей. Для этого:

Найти общий знаменатель;

Определить дополнительные множители для каждого члена уравнения;

Умножить числители дробей и целые числа на дополнительные множители и записать все члены уравнения без знаменателей (общий знаменатель можно отбросить);

Перенести слагаемые с неизвестными в одну часть уравнения, а числовые слагаемые - в другую от знака равенства, получив равносильное равенство;

Привести подобные члены;

Основные свойства уравнений

В любой части уравнения можно приводить подобные слагаемые или раскрывать скобку.

Любой член уравнения можно переносить из одной части уравнения в другую, изменив его знак на противоположный.

Обе части уравнения можно умножать (делить) на одно и то же число, кроме 0.

В примере выше для решения уравнения были использованы все его свойства.

Способы решения простых уравнений

Понятие уравнения.
Часто приходится встречаться с такой штукой, как уравнение. Что это такое надо знать. Но знать- это мало. Надо иметь хотя бы маленькое представление о том, как их решать. Посмотрим, что это такое.

Пусть у нас какое-нибудь число, к примеру х. Такой знак обычно ставят в уравнение и называют переменной. Положим, х=3. Дано выражение х+2=5. Это выражение- и есть простейшее уравнение, в котором надо найти, чему будет равен х. х- значение, или корень данного уравнения. корня может быть и 2, и 3, и сколько угодно, или вообще нет. Но в простейших всегда 1 корень.

Смысл решения уравнения.
Посмотрим, как же решать это уравнение. Часто надо понимать смысл. Дано уравнение х+1=7. Возьмите и начертите какую-нибудь прямую или линию, или просто представьте. На ней пусть отмечена точка 7, она же точка у(это тоже переменная, её тоже часто ставят. В данном случае х+1=у). Теперь сдвинем точку 7 назад на 1, т.е., она перейдёт в точку 6. Точно такое же значение примет у-1. Получим, что у-1=х+1-1=х. У нас х=6. Это и есть решение уравнения, или его корень.

Т.е., у уравнения есть 2 части, разделённые знаком равно. Мы, изменяя 1-ую часть, изменяем и вторую, т.е., получим:
В уравнении каждую его часть можно сладывать, вычитать, умножать, делить, возводить на 1 и то же число, а также бучиксировать.
Последние 2 действия нам не важны в решении простейших уравнений. Их используют при решении сложных.

В данном примере мы вычли из каждой части 1. Всё осталось равным. На самом деле, 6+1=7 и х+1=7, значит х и 6 одно и тоже. Такое преобразование называется равносильным. Так мы и поступаем во всех простых уравнениях с обычными арифметическими действиями. Рассмотрим примеры:
Полезные действия при решении уравнений.
1) 4+х=8 Отнимем от каждой части 4, т.е., 0+х=4 или х=4
2) х-5=2 Прибавим к обеим частям 5, получим х-5+5=2+5, х-0=7, х=7
3) х+1=х Надо такое число, складывая которое с 1, не изменится. Такого числа не существует, поэтому х не имеет корней
4) х+0=х Любое число, сложив с 0, не изменяется. Поэтому х является любым числом
5) 3-х=2 Вот это уже сложный пример. И хотя можно логически догадаться, мы решим так, как доказывает шаровую логику. Х под минусом. Поэтому тут немного сложнее. У нас 2 способа:
1\ Отнимем 3 от каждой части: 0-х=2-3=-1, или -х=-1(0-х=-х). Здесь можно использовать 2 способа, но мы выберем смысловой. -х и -1. У обоих есть минус. Т.е., значит, и х=1, мы просто убрали у них минусы, изменили их в другую сторону. На линии точка 0 и -1. 0=О, -1=А. Отрезок ОА мы развернём к +1. Это показывает то, что минусы можно отбрасывать, но если они есть у обоих частей.
Теперь мы посмотрим другой способ(второй тип первого способа заключался в том, что можно умножить обе части на -1, но мы до этого ещё не дошли): Сложим в каждом уравнении х: 3-х+х=2+х, 2+х=3, х=1
6) 2+х=3+х Сразу видно, х не имеет решений, и по смыслу, и так: 2+х-х=3+х-х, 2=3 что это? неверное равенство! Можно сделать вывод при решении простых уравнений: В уравнении можно перенести какое-нибудь слагаемое, изменив его знак на противоположный Например, х+4=6. Перенесём 4, изменив знак на противоположный, т.е. х=6-4=2. Противоположное для 4 число -4. Ставим или убираем минус. Мы так и поступали, но понимая под таким углом, решать легче. Попробуйте сами и вы в этом убедитесь.
7) х+5=15-х Перенесём -х в другую сторону, то есть 2х+5=15(знак умножения отбрасывают для сокращения). 2х=10, х=5(Почему так, это позже)

Уравнения с умножением и делением.
Рассмотрим простой пример:
1) 2х=10
Он был у нас недавно. Теперь мы объясним это. Мы можем обе части поделить на 2: 2х:2=10:2, х=5. В умножении всё аналогично сложению. Мы делаем то же самое. В уравнении можно перенести какой-нибудь множитель, изменив его знак на взаимно-обратный Как это понять? Например, перенеся 2 на ту сторону получим 1:2. 2:1 и 1:2- взаимно-обратны. Иногда 1: не обязательно. В 2х=10, 2 переносим, изменяя знак, получим- х=10х1:2. Мы просто поменяли знак. Если стоит знак деления, т.е., х:4, то мы переставим, поставив знак умножения.
2)х:6=12:6 переносим, изменяя знак на противоположный. Тогда, 12х6=72. х=72 Часто в уравнении важно не только умение решения, но и опыта при счёте
3)21162:х=705,4 Здесь надо использовать логические соображения. Как в сложении, можно х перенести к 705,4, получим новое уравнение 705,4х=21162, х=21162:705,4=30. Не надо бояться чисел и уравнения. Например, уравнение большое, но на самом деле, оно такое лёгкое, надо просто его решить. Или например большие числа. Замените их на мелкие числа, вы сразу поймёте как решать. Потом замените на исходные и сосчитайте. Если уж вообще тяжело, воспользуйтесь калькулятором.
4) х+х+5+х+4+х+х+5+х+х+х+6+1+х=102 Здесь мы просто соединим иксы и числа:х+х+х+х+х+х+х+х+х+5+5+4+6+1=9х+21 Дальше, 21 перенесём, 102-21=81, получим 9х=81, х=81:9=9
Теперь рассмотрим ещё один пример:
5)20х-6=51+12 Сложим 51 и 12, 51+12=63. Теперь 6 перенесём, 63+6=69. х=69:20. Но 69 на 20 не делится. Поэтому нам можно оставить так, но лучше, 690:2:100=345:100=3,45. :100 мы определили по логическим соображениям.
6)4:х=2хПеренесём:х на ту сторону, получим, 2хх=4, х на х=2. В таком случае, ответ будет корень из 2, но этого вам пока не надо:
Ответ: корень из 2

Упрощение переноса.
Возьмём к примеру, уравнение а+х=б. В таком случае, «а» переносим на ту сторону, получаем х=б-а. То же самое мы могли сделать, чтобы найти а. Другой пример: х-а=б. Тогда а переносим на ту сторону, т.е., х=б+а. Если а-х=б, то мы х можем перенести на ту сторону, т.е, а=х+б. Это мы рассмотрели. Теперь уберём б, тогда, х=а-б.
В умножении и делении рассуждения аналогичны. Чтобы найти слагаемое, надо отнять от суммы другое(другие) слагаемые. (Например, 3+х=6. 3- другое слагаемое, поэтому от суммы 6 отнимаем 3)
Чтобы найти уменьшаемое, надо сложить все остальные числа. (Например, х-6=3. 6 и 3 складываем, так как они- остальные числа)
Чтобы найти вычитаемое, надо от уменьшаемого отнять разность. (Например, 6-х=3. 6- уменьшаемое, 3- частное. Поэтому, х=6-3)

Так же дела обстоят, когда много чисел. К примеру, 5-х-у+3=12. Чтобы найти х, а это вычитаемое, надо сначала найти уменьшаемое. Это не 5, как многие думают. Объединим всё в 1 кучу, т.е., (5+3-у)-х=12, х=5+3-у-12 Кстати, нахождение вычитаемого самое сложное, но вы к этому привыкните.

1) х:3у=12. Чтобы найти х, надо всё остальное перемножить. Это как сложение, просто мы меняем знаки действий аналогично: х=3у Х 12= 36у.
2) 2у:(х+1)=4м х+1- это как один х, но с зависимыми числами, по типу причастного или деепричастного оборота. Найти оборот можно как обычно: х+1=2у:4м, х=0,5у:м-1 (Мы здесь сократили. Желательно сокращать, где можно, так ведь легче решать)Раскрытие скобок и вынос за скобки
Мы уже решали, переносили. Но иногда приходится сталкиваться с другими проблемами решения уравнений.
1) 4+(х-5)=12 Если перед скобками +, то скобки можно опустить:
4+х-5=12-1+х=12х=13
Хоть здесь и надо было решить не обязательно так. Но это мы сделали ради примера. Но если стоит минус:4-(х-5) Тогда мы тоже раскрываем, но знаки внутри скобок станут противоположными:4-х+5 Почему так происходит? Это надо разбирать. Пусть у нас 12-(3+5)=4. Мы будем поочерёдно вычитать, сначала 12-3, потом 12-3-5, таким образом, мы раскрыли скобки. А если 12-(3-5)=14? Тогда мы можем прибавить к обеим частям (3-5). У нас получится:12=14+(3-5). Тогда мы просто уберём: 14+3-5 и получим верное равенство. Это получается из-за переноса и перемены знак на противоположный. С другой стороны, при 12-(3-5). Мы можем сначала прибавить 5, это даже по смыслу понятно, 3-5+5. Потом остаётся отнять 3: 12+5-3. Но это то же самое, что и 12-3+5. Таким образом, в этом нетрудно разобраться. Это действительно и при многих числах. К примеру, -(х+у-2+4+6-2а+3б)= -х-у+2-4-6+2а-3б. Решим к примеру:
2) 5+х-(х+2)=2+х Это легко сделать, раскрыв скобки:5+х-х+2=2+х2+х=7, х=5

Таким образом, у нас свойства:
1) От перестановки слагаемых сумма не меняется(при пересмтановки множителей тоже)
2) При раскрытии скобок с вычитанием все знаки в скобках меняются на противоположные(при раскрытии деления то же самое, только меняется на взаимно обратные) Теперь познакомимся с такой вещью, как распределительное свойство. К примеру, как решить 5х-2х=12? В таком случае, приводят подобные слагаемые, т.е., коэффициенты 5 и 2 объединяются:(5-2)х=12

Как это сделали? Удивительно? Но это практически самое основное правило математики. На нём держатся практически все задачи. Рассмотрим. У нас 2 группы бутылок в 2 ряда. В 1 группе 5 штук, во второй, 3. Но мы можем вторую группу подставить к первой, тогда у нас будет 8 бутылок в 2 ряда. Но это и есть то самое свойство:5+5+3+3. По первому свойству, поменяем слагаемые: 5+3+5+3= (5+3)+(5+3). Вот и всё.

3) Распределительное свойство умножения- ах+бх= (а+б)х и наоборот3) 3(4+х)+5(4+х). Сократить:(3+5)(4+х)= 8(4+х)= 32+8х Таким образом, мы ещё облегчили решение уравнений.Линейные уравнения Мы рассмотрели многие свойства и преобразования. Теперь покажем общий вид уравнений, с которыми часто сталкиваются, и их придётся решать.
Это базовая основа. Линейные уравнения вида ах+б=0 или ах+б=сх+д Покажем примеры:
1) 4х+12=20 Переносим 12 или по свойству: 4х=20-12=8, х=2
Таким образом, решение уравнения ах+б=с таково: х=(с-б):а
2) 12-40х=25 Поставим так: -40х+12=25, теперь х= (25-12):(-40)= -13:40=-0,325
3) 5х+2=7х-7 Здесь желательно переносить с иксами на 1 сторону, с числами- на другую, чтобы сокращать. Лучше всё делать по очереди и переносить так, чтобы избежать отрицательных чисел.2=7х-5х-7=2х-7, далее -7: 2+7=2х, 2х=9, х=4,5

Задачи.
Часто в задачах всё решается через уравнения. Любая задача- это своего рода уравнения, корни которого- какая-то величина.
1) Вася за 3 дня вспахал на 6 аров меньше, чем за 5 дней. Найти, сколько вспахал. С первого взгляда кажется, что задача нерешима, т.е., в ней недостаточно данных. На самым деле, нужно просто уметь составлять математическую модель. Пусть х- это вспахано у Васи: 5х и 3х. 3х меньше 5х на 5, т.е., 3х+5=5х. Решаем это уравнение и получим, х=2,5 аров. Задача решена.
2) У Васи марок на 10 больше, чем у Пети. Но вместе у них 40 марок. Найти, сколько марок у каждого. Пусть у Пети х марок, тогда у Васи х+10, т.е., на 10 больше. Вместе, т.е., х+(х+10)=40, решаем соответствующее уравнение: 2х=30, х=15- это у Пети. У Васи 15+10=25 Иногда приходится сталкиваться с большими количествами переменных, но и там часто применяются линейные методы. Здесь мы это не будем рассматривать.
3) У Васи и у Пети 30 машинок. Но у Сени тоже есть машинки, и если Вася отдаст Сене 5 машинок, то у Сени будет машинок вдвое больше, чем у Васи. Но если Петя отдаст ещё 5 машинок, то тогда у Сени будет втрое больше, чем у Васи. Найти, сколько машинок у каждого. Создадим несколько переменных: х-Вася, у-Петя, а-Сеня. Тогда получится система, в которой нужно найти общие решения.х+у=30а+5=2(х-5)а+5+5=3(х-5) В таком случае выражают 1 переменную через другую и решают уравнения. Но иногда применяются и другие методы. Видим, что с прибавлением 5 к Сене, у нас получилось прибавление и х-5. Тогда, 5=х-5, а х=10. у=30-10=20. Итак, у Васи 10, у Пети 20. Сеню найти легко, подставив значения. а+5=2(х-5). х-5=5, тогда: а+5=2Х5=10, а=5Ответ: у Васи 10, у Пети 20, у Сени 5. Теперь посмотрим ещё 1 сложный вариант:
4) Сумма цифр трёхзначного числа 9. Если убрать последнюю цифру, и поменять цифры местами в оставшемся двузначном числе, то получится, что оно на 9 меньше, чем прежнее двузначное. А если убрать первую цифру, а остаток тоже поменять местами, то получится на 45 больше. Найти это число. Попробуйте решить эту задачу самостоятельно. Если вы сможете, то вы уже умеете хорошо решать уравнения и составлять математическую модель. Но вы, в принципе, можете посмотреть, как решать. Пусть х,у,з- это цифры. Тогда, у нас снова наподобие системы, получим данные:х+у+з=9ух+9=хууз+45=зу Начать можно методом пушики. Мы будем подбирать такие числа, что ух+9=ху. У нас есть:12 и 21, 23 и 32, 34 и 43, 45 и 54 и т.д. Мы заметили, что разница между цифрами в 1, т.е., 1+1=2 и 2-1=1 и т.п. Из этого можно заменить у, как х-1,т.е.,х+х-1+з=9, 2х+з=10Теперь посмотрим возможные варианты с плюсованием 45. Для этого втора цифра больше первой, имеем: 16 и 61, 27 и 72, 38 и 83, 49 и 94. Из этих вариантов следует, что вторая цифра на 5 больше, т.е., у+5=з., но у=х-1. Мы получили, что з=х-1+5=х+4. Тогда:2х+х+4=10, 3х=6, х=2. х-1=1, х+4=6. Получаем число 216. Ответ: 216

Линейные неравенства.
В заключении, покажем, что же такое линейные неравенства. Это похоже на уравнение, но х тут меньше или больше чего-то. В неравенствах действуют такие же принципы, что и в уравнениях. Обе части можно складывать, умножать, возводить и т.п. К примеру:
1)x+4 4х-2Здесь мы можем получить, что 5х+4>4х, и х+4>0. Переносим и получаем, что х больше -4 В неравенствах действуют все свойства линейных уравнений Надо учитывать, что есть и сложные неравенства, которые решаются по другому. Также, как и уравнения, неравенства могут не иметь решений, или иметь любые решения.
3)х+4 х Ещё один интересный случай. Заметим, что если перенести х на ту часть, то получится, что х больше нуля.
5)аХа

Решение простых уравнений. 5 класс

Уравнение - это равенство, содержащее букву, значение которой надо найти.

В уравнениях неизвестное обычно обозначается строчной латинской буквой. Чаще всего используют буквы « x » [икс] и « y » [игрек].

  • Корень уравнения - это значение буквы, при котором из уравнения получается верное числовое равенство.
  • Решить уравнение - значит найти все его корни или убедиться, что корней нет.

Решив уравнение, всегда после ответа записываем проверку.

Информация для родителей

Уважаемые родители, обращаем ваше внимание на то, что в начальной школе и в 5 классе дети НЕ знают тему «Отрицательные числа».

Поэтому они должны решать уравнения, используя только свойства сложения, вычитания, умножения и деления. Методы решения уравнений для 5 класса приведены ниже.

Не пытайтесь объяснить решение уравнений через перенос чисел и букв из одной части уравнения в другую с изменением знака.

Освежить знания по понятиям, связанным со сложением, вычитанием, умножением и делением вы можете в уроке «Законы арифметики».

Решение уравнений на сложение и вычитание

Как найти неизвестное
слагаемое

Как найти неизвестное
уменьшаемое

Как найти неизвестное
вычитаемое

Чтобы найти неизвестное слагаемое, надо от суммы отнять известное слагаемое.

Чтобы найти неизвестное уменьшаемое, надо к разности прибавить вычитаемое.

Чтобы найти неизвестное вычитаемое, надо от уменьшаемого отнять разность.

x + 9 = 15
x = 15 − 9
x = 6
Проверка

x − 14 = 2
x = 14 + 2
x = 16
Проверка

16 − 2 = 14
14 = 14

5 − x = 3
x = 5 − 3
x = 2
Проверка

Решение уравнений на умножение и деление

Как найти неизвестный
множитель

Как найти неизвестное
делимое

Как найти неизвестный
делитель

Чтобы найти неизвестный множитель, надо произведение разделить на известный множитель.

Чтобы найти неизвестное делимое, надо частное умножить на делитель.

Чтобы найти неизвестный делитель, надо делимое разделить на частное.

y · 4 = 12
y = 12: 4
y = 3
Проверка

y: 7 = 2
y = 2 · 7
y = 14
Проверка

8: y = 4
y = 8: 4
y = 2
Проверка

Уравнения 5 класса

Сегодня мы рассмотрим более сложные уравнения 5 класса, содержащие несколько действий. Чтобы найти неизвестную переменную, в таких уравнениях надо применить не одно, а два правила.

1) x:7+11=21

Выражение, стоящее в левой части - сумма двух слагаемых

Таким образом, переменная x является частью первого слагаемого. Чтобы найти неизвестное слагаемое, надо из суммы вычесть известное слагаемое:

Получили простое уравнение 5 класса, из которого надо найти неизвестное делимое. Чтобы найти неизвестное делимое, нужно частное умножить на делитель:

2) 65-5z=30

Правая часть уравнения представляет собой разность:

Переменная z является частью неизвестного вычитаемого. Чтобы найти неизвестное вычитаемое, надо из уменьшаемого вычесть разность:

Получили простое уравнение, в котором z - неизвестный множитель. Чтобы найти неизвестный множитель, надо произведение разделить на известный множитель:

3) 120:y-23=17

В правой части уравнения - разность. Переменная y является частью неизвестного уменьшаемого.

Чтобы найти неизвестное уменьшаемое, надо к разности прибавить вычитаемое:

Здесь y - неизвестный делитель. Чтобы найти неизвестный делитель, надо делимое разделить на частное:

4) (48+k) ∙ 8=400

Левая часть уравнения представляет собой произведение. Переменная k - часть первого множителя:

Чтобы найти неизвестный множитель, надо произведение разделить на известный множитель:

В новом уравнении k - неизвестное слагаемое:

Здесь мы решали уравнения 5 класса без использования свойств сложения и вычитания. В 6 классе правила раскрытия скобок упрощаются, и решать такие уравнения становится проще.

182 Comments

Спасибо огромное самый лучший сайт где я искала уравнения

Спасибо Вам за труды! Все изложено настолько доступно, что мой сын сказал, что Вы «классный» учитель. Простите за цитату, но прочитав Ваши разъяснения, он все понимает. Хотя до этого, в 5 классе, все это проходил, но недопонимал.

Спасибо Вам, Наталья, за теплые слова!

как решить x(x+4)=77

В 5 классе я могу только посоветовать угадать корни этого уравнения. Можно рассуждать так: 77=7х11. Поэтому один из множителей должен равняться 7, другой - 11. Поскольку х+4 больше, чем х, то х=7.
Позже Вы узнаете, что это уравнение - квадратное, и корней у него два. Второй корень - число отрицательное, в 5 классе их еще не учат. (Второй корень х=-11).

как решить такое уравнение??144-(х:11+21)*5=14 спасибо

144 - уменьшаемое, (х:11+21)*5 - вычитаемое, 14 - разность. х - элемент неизвестного вычитаемого. Чтобы найти неизвестное вычитаемое, надо из уменьшаемого вычесть разность: (х:11+21)*5=144-14, отсюда (х:11+21)*5=130. В новом уравнении х:11+21 - 1й множитель, 5 - 2й множитель, 130 - произведение. х - элемент неизвестного первого множителя. Чтобы найти неизвестный множитель, надо произведение разделить на известный множитель: х:11+21=130:5, отсюда х:11+21=26. В новом уравнении х:11 - 1-е слагаемое, 21 - 2-е слагаемое, 26 - сумма. х - элемент 1-го слагаемого. Чтобы найти неизвестное слагаемое, надо из суммы вычесть известное слагаемое: х:11=26-21, х:11=5. В этом уравнении х - делимое, 11 - делитель, 5 - частное. Чтобы найти неизвестное делимое, надо делитель умножить на частное: х=5∙11, х=55. Ответ: 55.
Полезно проверить себя: 144-(55:11+21)∙5=144-(5+21)∙5=144-26∙5=144-130=14. Верно.

Я закінчила 5 клас. Мені 11 років. І мені дуже подобається розв’язувати рівняння. Я розв’язала всі рівняння які давали Вам і в мене все вийшло як і у Вас. Дякую.

помогите решить 4x-x=8.7

Приводим подобные слагаемые в левой части уравнения:
3х=8,7
Обе части уравнения делим на число, стоящее перед иксом:
х=8,7:3
х=2,9

как решить такое уравнение:
(5.4у + 8.3) * 2.1= 23.1

(5,4у + 8,3) * 2,1= 23,1
(5,4у + 8,3) - неизвестный множитель. Чтобы найти неизвестный множитель, надо произведение разделить на известный множитель:
5,4у + 8,3 = 23,1:2,1
5,4у + 8,3 =11
Чтобы найти неизвестное слагаемое 5,4y, надо из суммы вычесть известное слагаемое:
5,4у=11-8,3
5,4у=2,7
Чтобы найти неизвестный множитель, надо произведение разделить на этот множитель:
у=2,7:5,4
у=0,5
При решении уравнений с десятичными дробями удобно сначала избавиться от запятой. Как это сделать, постараюсь рассказать на днях.

У меня такая же проблема. Только там где стоит умножение у меня вычитание

Как решить вот это уравнение?
(5.4у + 8.3) - 2.1 = 23.1

Я считаю что там где стоит ‘вычитание’ должно быть ‘умножение’
Задание на печатала сама учительница, поэтому всё должно быть правильно. Но решить не получается.
Помогите пожалуйста, заранее спасибо

(5.4у + 8.3) - 2.1 = 23.1
Ищем неизвестное уменьшаемое:
5.4у + 8.3 = 23.1 + 2.1
5.4у + 8.3 = 25.2
Теперь найдем неизвестное слагаемое:
5.4у = 25.2 - 8.3
5.4у =16.9
Осталось найти неизвестный множитель:
y=16.9/5/4
y=169/54
и выделить из неправильной дроби целую часть
y=3 7/54

Помогите решить:
14y-2y+76=100

Степан, 14y и 2y - подобные слагаемые. Значит, их можно вычесть: 14y-2y=12y.
Тогда в уравнении 12y+76=100 12y - неизвестное слагаемое. Найдите 12y как неизвестное слагаемое. После этого в произведении 12y ищите y как неизвестный множитель.

Алина, сумму в правой часто можно найти: (18-х)+10=56
Между скобками и 10 стоит «+», значит, выражение в скобках - неизвестное слагаемое: 18-х=56-10; 18-х=46. Остается найти неизвестное вычитаемое х: х=18-46; х=-28.

Выражение в скобках, 5x-7 - делитель. Чтобы найти неизвестный делитель, надо делимое разделить на частное: 5x-7=528:16; 5x-7=33. 5x - уменьшаемое. Чтобы найти неизвестное уменьшаемое, надо к разности прибавить вычитаемое: 5x=33+7; 5x=40. Остается найти неизвестный множитель: x=40:5; x=8.

как решить такое уровнение 11у+32у-127=45

Сначала нужно привести подобные слагаемые: 11у+32у-127=45; 43y-127=45. 43y - неизвестное уменьшаемое. Чтобы найти неизвестное уменьшаемое, нужно к разности прибавить вычитаемое: 43y=45+127; 43y=172. Чтобы найти неизвестный множитель y, нужно произведение разделить на известный множитель: y=172:43; y=4.

спасибо вам, Светлана.

День добрый. Помогите пожалуйста решить уравнение (9x+7)*y=45x+y. Спасибо!

Сергей, это уравнение - с двумя переменными (x и y). Нужно или еще одно уравнение (чтобы количество неизвестных было не больше количества неизвестных), либо какие-либо дополнительные условия.

Помогите как решать подобные уравнения - 7х-26,7-2х.ну так, к примеру, а то нигде нет. Заранее спасибо. сайт очень полезный

Даша, это уравнение - с подобными слагаемыми. Постараюсь написать отдельный пост по решению таких уравнений.
P.S. Здесь:http://www.for6cl.uznateshe.ru/uravneniya-s-podobnymi-slagaemymi/

помогите как решить это уравнение 10x+x+1=4*(x+x+1)

Это - линейное уравнение.
Сначала следует привести подобные слагаемые:11x+1=4*(2x+1). Затем - раскрыть скобки: 11x+1=8x+4. Теперь неизвестные переносим в одну сторону, известные - в другую, изменив при этом их знаки:11x-8x=4-1. Упрощаем:3х=3. Теперь обе части уравнения делим на число, стоящее перед иксом: х=3:3, х=1.

не могу понять, Светлана Иванова, помогите..5(14+b)+6b=158… вроде делаю как Вы изложили, да видать не усвоил))) распишите еще раз)))

Аскар, сначала раскройте скобки: 70+5b+6b=158. Это - уравнение с подобными слагаемыми, как раз недавно речь о таких уравнениях велась. После приведения подобных слагаемых получаем 70+11b=158. А дальше - все, как обычно: 11b - неизвестное слагаемое, 11b=158-70, 11b=88. b - неизвестный множитель, b=88:11? b=8.

Как решить такое уравнение: (19*700):70+(850+х)=6000:50 Заранее спасибо!

Сначала уравнение надо упростить: 19*(700:70)+(850+х)=6000:50; 19*10+(850+х)=120; 190+(850+х)=120.Здесь можно пойти двумя путями: либо раскрыть скобки, либо выражение в скобках рассматривать как неизвестное слагаемое. Например, 190+850+х=120;
1040+x=120;x=120-1040; х=-920.

Здравствуйте! А как решить x ÷ 9 = x ÷ 5? Если не сложно?!)

Это линейное уравнение. Неизвестные слагаемые переносим в одну сторону, известные - в другую, изменив при этом их знаки: x-x=5-9; 0x=-4. Это уравнение не имеет корней.

Ваше решение правильное (если уже прошли дроби). Вариант с использованием основного свойства пропорции: 5х=9х; 5х-9х=0; -4х=0, х=0 - легче, но пропорцию еще не учили.

помогите, пожалуйста как решить эту задачу,
заранее спасибо!
Паук и муха сидят на противоположных вершинах куба. Паук может ползти по ребру куба и по диоганали грани куба. Сколько существует вариантов движения паука к мухе?

Здравствуйте. Светлана помогите решить эту задачу, если не сложно.
Паук и муха сидят на противоположных вершинах куба. Паук может ползти по ребру куба и по диагоналиграни куба. Сколько существует вариантов движения паука и мухе?

Здравствуйте помогите разобрать уравнение 5а + 5 *14= 8 * м - 8 *15

Алексей, уточните, пожалуйста условие. У Вас в условии 2 переменные.

Помогите пожалуйста решить!
9(143-13х)=234

Между 9 и выражением в скобках стоит знак «∙» (хотя его не пишут). Значит, левая часть - это произведение. Чтобы найти неизвестный множитель (143-13х), надо произведение разделить на известный множитель: 143-13х=234:9;143-13х=26.
143-13х - разность. Чтобы найти неизвестное вычитаемое 13х, надо из уменьшаемого вычесть разность:13х=143-26;13х=117.
13х - произведение. Чтобы найти неизвестный множитель x, произведение делим на известный множитель: х=117:13; х=9.

Помогите решить- 88000:110+x=809

Упрощаем: 800+x=809 и находим неизвестное слагаемое x=809-800,x=9.

Помогите не могу решить уравнение 5-х*х=1
Надо срочно!

Помогите решить уравнение (надо очень срочно) 5-х*х=1

5-x²=1. Здесь x² - неизвестное вычитаемое. Чтобы его найти, надо из уменьшаемого вычесть разность:x²=5-1, x²=4. Квадрат какого числа равен 4? 2. Если уже прошли отрицательные числа, то еще и -2. То есть x=2 и x=-2.

Здравствуйте, помогите пожалуйста решить уравнение 5(а-2)+3(а+3)

Здравствуйте, Ангелина! Вы забыли указать, чему равно это выражение.

помогите решить уравнение 13(х+6)-72=123

13(х+6) - неизвестное уменьшаемое. Чтобы его найти, нужно к разности прибавить вычитаемое: 13(х+6)=123+72, 13(х+6)=195.Теперь ищем неизвестный множитель (х+6). Для этого надо произведение разделить на известный множитель:х+6=195:13, х+6=15. Осталось найти неизвестное слагаемое x=15-6, x=9.

Это уравнение в 5 классе? В 6 классе я бы посоветовала умножить обе части уравнения на 7. Получаем 7x+x=224∙7, 8x=1568, x=1568:8, x=196.

(8Х+24):5:4+6- неизвестный делитель, следовательно, делимое делим на частное: (8Х+24):5:4+6=10:1, (8Х+24):5:4+6=10.
(8Х+24):5:4 - неизвестное слагаемое, из суммы вычитаем известное слагаемое: (8Х+24):5:4=10-6, (8Х+24):5:4=4.
(8Х+24):5 - неизвестное делимое, следовательно, частное умножаем на делитель: (8Х+24):5=4∙4, (8Х+24):5=16.
Далее ищем неизвестное делимое: 8Х+24=16∙5, 8Х+24=80; неизвестное слагаемое 8Х=80-24, 8Х=56; и неизвестный множитель:
x=56:8, x=7.

Условие было таким.Одно из чисел в 7 раз меньше другого. Найдите эти числа, если их сумма равна 224? Это задача 5 класса.

Ольга, при решении задач всегда лучше брать за x то, что меньше. В Вашей задаче примем за x меньшее число, тогда большее - 7x. Так как их сумма равна 224, имеем уравнение: 7x+x=224, 8x=224, x=224:8, x=28.
Значит, меньшее число рано 28, а большее - 7∙28=196.
Как видите, так проще.

Помогите решить уравнение, пожалуйста!

97+75:(50-5х)=300:3, 97+75:(50-5х)=100,
75:(50-5х)=100-97, 75:(50-5х)=3,
50-5х=75:3,50-5х=25,
5х=50-25,5х=25,
х=25:5, х=5.

Спасибо Вам огромное, Светлана Ивановна! В жизни бы не догадалась, как поступить проще.

Пожалуйста, Ольга!
Только Светлана Иванова?

Помогите решить уравнение 2х+8+4х=20

помогите решить уравнение 4 целых 2 девятых + (16 целых 5 девятых - x) = 15 целых 1 девятая - 8 целых 7 девятых

4 2/9 +(16 5/9 - x)=15 1/9 - 8 7/9
15 1/9 - 8 7/9=14 10/9 - 8 7/9=6 3/9.
4 2/9 +(16 5/9 - x)=6 3/9
16 5/9 - x=6 3/9 - 4 2/9
16 5/9 - x=2 1/9
x=16 5/9 - 2 1/9
x=14 4/9

здравствуйте помогите решить уравнение (2х-200):13-1=123

и пожалуйста ещё уравнение очень нужно помогите (321+х)45-85=77

(321+х)∙45-85=77
(321+х)∙45=77+85
(321+х)∙45=162
321+х=162:45
321+х=3,6
х=3,6-321
х=-317,4

(2х-200):13-1=123
(2х-200):13=123+1
(2х-200):13=124
2х-200=124∙13
2х-200=1612
2х=1612+200
2х=1812
x=1812:2
х=906

помогите решить уравнение (476-х):31=320:31

(476-х):31=320:31
476-х=320
х=475-320
х=155

как объяснить ребенка переход от первой строки ко второй? куда исчезло деление на 31?

Два числа разделили на одно и то же число 31, получили равные результаты. Следовательно, эти числа равны между собой.

Здравствуйте,Светлана.Помогите пожалуйста решить уравнение. 123+у=357- 85

123+у=357- 85
123+у=272
у=272-123
у=149
Антон, это уравнение Вы вполне могли бы решить самостоятельно. Все необходимые подсказки и пояснения на сайте есть. Постарайтесь разобраться.

Помогите решить такое уравнение:
7.5x-2.46x=78.3+124.56

Сначала упрощаем обе части уравнения:
5,04x=202,86
Затем ищем неизвестный множитель:
x=202,86:5,04
x=20286:504
x=40,25

Помогите решить уравнение
2,4x+x+9,1=38

Сначала упрощаем левую часть уравнения
3,4х+9,1=38. Затем ищем неизвестное слагаемое:3,4х=38-9,1; 3,4х=28,9. Затем - неизвестный множитель: х=28,9:3,4; х=8,5.

Светлана добрый день. Читал Ваши коментарии, очень понравилось как Вы объясняете. Объясните пожалуйста как решить задачу и составить уравнение по ней: во дворе находятся куры и ягнята. Известно, что ягнят в три раза меньше, чем кур. Количество ног кур и ягнят составляет 40. Сколько во дворе кур и сколько ягнят? Заранее спасибо.

Нурлан, здравствуйте!
Пусть во дворе х ягнят, тогда кур - 3х. У каждого ягненка 4 ноги, значит, у всех ягнят ног 4х. У каждой курицы 2 ноги, поэтому у всех кур ног 3х∙2=6х. Всего ног у кур и ягнят 4х+6х, что по условию задачи равно 40. Составим и решим уравнение:4х+6х=40; 10х=20; х=4. Значит, во дворе 4 ягненка и 3∙4=12 кур.

как решить такое уравнение? 27(n-27)=27?

27(n-27)=27
Чтобы раскрыть неизвестный множитель, надо произведение разделить на известный множитель:
n-27=27:27
n-27=1. Чтобы найти неизвестное уменьшаемое, надо к вычитаемому прибавить разность:
n=27+1
n=28.

Светлана добрый день, помогите пожалуйста объяснить ребенку в пятом классе как решить задачу: Чашка кофе с сахаром стоит 1.10$, кофе стоит дороже сахара на 1$, сколько стоит сахар. Вот проблема в том что уравнения с двумя неизвестными они еще не проходили.

Извините, не всегда получается ответить вовремя, увы.
Пусть сахар стоит x $, тогда кофе - (x+1) $. Следовательно, чашка кофе с сахаром стоит x+(x+1) $, что по условию задачи равно 1.10 $ Составляем уравнение и решаем его:
x+(x+1)=1,1
x+x+1=1,1
2x=1,1-1
2x=0,1
x=0,1:2
x=0,55
Значит, сахар стоит 0,55 $. Если десятичные дроби еще не проходили, нужно цены сразу же перевести в центы.

Как решить уравнения 29х-15х+16=100
Пожалуйста помогите

14х+16=100
14х=100-16
14х=84
x=84:14
x=6.

www.for6cl.uznateshe.ru

Решение уравнений

На данном уроке подробно рассмотрены способы решения уравнений. Объяснены способы решения уравнений, как методом подбора, так и с учетом взаимосвязи компонентов действий сложения и вычитания.

Если у вас возникнет сложность в понимании темы, рекомендуем посмотреть урок «Уравнения и неравенства»

Введение понятия «уравнение»

Определим, что такое «уравнение».

Правильный ответ: уравнение – это математическое равенство, которое содержит неизвестное число. Неизвестное число обозначают буквами латинского алфавита.

Найдем среди данных записей уравнения.

первая запись – это равенство, но в нем отсутствуют буквы латинского алфавита, значит, она не является уравнением;

вторая запись – это неравенство, поэтому не соответствует определению уравнения;

третья запись – это математическое равенство, которое содержит неизвестное число, обозначенное буквой латинского алфавита, значит, является уравнением;

четвертая запись не является равенством, значит, это не уравнение.

Введение понятия «корень уравнения»

Что значит «решить уравнение»?

Правильный ответ: решить уравнение – значит найти такое числовое значение неизвестного, при котором равенство будет верным.

В математике говорят: решить уравнение – это значит найти корень уравнения.

Решение уравнение способом подбора

Из чисел 2, 5, 8, 11 выберем для каждого уравнения такое значение х, при котором получится верное равенство.

В первое уравнение 18-х =10 подставим первое число 2. Получаем: 18-2=10. Это равенство нельзя назвать верным. Значит, число 2 не является корнем данного уравнения. Подставим в это уравнение число 5. Получаем: 18-5=10. Это равенство также нельзя назвать верным. Значит, число 5 тоже не является корнем данного уравнения. Подставим в это уравнение число 8. Получаем: 18-8=10. Это равенство можно назвать верным. Значит, число 8 является корнем данного уравнения.

Продолжаем рассуждать. В уравнение 2 + х = 7 подставим первое число 2. Получаем: 2+2=7. Это равенство нельзя назвать верным. Значит, число 2 не является корнем данного уравнения. Подставим в это уравнение число 5. Получаем: 2+5=7. Это равенство можно назвать верным. Значит, число 5 является корнем данного уравнения.

2-9=2, но 2 меньше, чем 9, поэтому вычитание мы выполнить не сможем. Нужно попробовать подставить в уравнение число, которое больше, чем 9. подставим число 11. Получаем: 11-9=2. Это равенство можно назвать верным. Значит, число 11 является корнем данного уравнения.

Найдем корень последнего уравнения. Подставим число 2 в уравнение х+8=10. Получаем: 2+8=10. Это равенство можно назвать верным. Значит, число 2 является корнем данного уравнения.

Данные уравнения мы решали способом подбора. Это способ не всегда бывает удобным. Уравнения можно решать и другим способом, но для этого нужно знать, как связаны между собой компоненты действий при сложении и вычитании.

Решение уравнений на основе знаний связи компонентов действий сложения и вычитания

Проверим себя. Как найти неизвестные компоненты?

а) чтобы найти неизвестное слагаемое, надо из суммы вычесть известное слагаемое.

б) чтобы найти неизвестное вычитаемое, надо из уменьшаемого вычесть значение разности.

в) чтобы найти неизвестное уменьшаемое, надо к значению разности прибавить вычитаемое.

Обратим внимание: если мы умеем находить слагаемые, уменьшаемое и вычитаемое, можно решать уравнения другим способом.

Решим уравнения с объяснением.

Рассуждаем так. В уравнении 64 + d =82 выполняется сложение. В уравнении известно первое слагаемое – 64 и значение суммы – 82. Неизвестно второе слагаемое. Вспомним правило: чтобы найти неизвестное слагаемое, надо из суммы вычесть известное слагаемое. Запишем.

Корень уравнения – 18. Проверим: 64+18=64+10+8=82. 82=82. Это верное равенство. Делаем вывод: если равенство верное, значит, уравнение решено правильно.

В уравнении b — 36 = 40 выполняется вычитание. В уравнении известно вычитаемое – 36 и значение разности – 40. Неизвестно уменьшаемое. Вспомним правило: чтобы найти неизвестное уменьшаемое, надо к значению разности прибавить вычитаемое. Запишем.

Корень уравнения – 76. Проверим: 76-36=76-30-6=40. 40=40. Это верное равенство. Делаем вывод: если равенство верное, значит, уравнение решено правильно.

В уравнении 82 — k = 5 выполняется вычитание. В уравнении известно уменьшаемое – 82 и значение разности – 5. Неизвестно вычитаемое. Вспомним правило: чтобы найти неизвестное вычитаемое, надо из уменьшаемого вычесть значение разности. Запишем.

Корень уравнения – 77. Проверим: 82-77=82-70-7=5. 5=5. Это верное равенство. Делаем вывод: если равенство верное, значит, уравнение решено правильно

Решение уравнений, соответствующих предложенной схеме

Выберем уравнения, которые соответствуют схеме, и найдем числовое значение х (рис. 1).

Рис. 1. Иллюстрация к заданию

Будем рассуждать. На данной схеме мы видим целое – 16, части – 2 и х.

Попробуем подобрать уравнение.

Рассмотрим уравнение х-2=16. В этом уравнении х – уменьшаемое, то есть самое большое число. Но на схеме самое большое число – 16, значит, это уравнение для данной схемы не подходит.

Рассмотрим второе уравнение 2+х=16. Видим, что 2 – это первое слагаемое, х – второе слагаемое. Из двух слагаемых получается целое – 16. Делаем вывод: данное уравнение к схеме подходит.

Решим его, найдем корень уравнения. Неизвестно второе слагаемое. Вспомним правило: чтобы найти неизвестное слагаемое, надо из суммы вычесть известное слагаемое. Запишем.

Рассмотрим третье уравнение 16-х=2. На схеме видим, что уменьшаемое 16 – это целое, х – вычитаемое (одна часть), 2 – значение разности (вторая часть). Делаем вывод: данное уравнение к схеме подходит.

Решим его, найдем корень уравнения. Вспомним правило: чтобы найти неизвестное вычитаемое, надо из уменьшаемого вычесть значение разности. Запишем.

Сегодня на уроке мы решали уравнения способом подбора и на основе знания связи компонентов действий при сложении и вычитании.

Список литературы

В этом видео мы разберём целый комплект линейных уравнений, которые решаются по одному и тому же алгоритму — потому и они и называются простейшими.

Для начала определимся: что такое линейное уравнение и какое их них называть простейшим?

Линейное уравнение — такое, в котором присутствует лишь одна переменная, причём исключительно в первой степени.

Под простейшим уравнением подразумевается конструкция:

Все остальные линейные уравнения сводятся к простейшим с помощью алгоритма:

  1. Раскрыть скобки, если они есть;
  2. Перенести слагаемые, содержащие переменную, в одну сторону от знака равенства, а слагаемые без переменной — в другую;
  3. Привести подобные слагаемые слева и справа от знака равенства;
  4. Разделить полученное уравнение на коэффициент при переменной $x$ .

Разумеется, этот алгоритм помогает не всегда. Дело в том, что иногда после всех этих махинаций коэффициент при переменной $x$ оказывается равен нулю. В этом случае возможны два варианта:

  1. Уравнение вообще не имеет решений. Например, когда получается что-нибудь в духе $0\cdot x=8$, т.е. слева стоит ноль, а справа — число, отличное от нуля. В видео ниже мы рассмотрим сразу несколько причин, по которым возможна такая ситуация.
  2. Решение — все числа. Единственный случай, когда такое возможно — уравнение свелось к конструкции $0\cdot x=0$. Вполне логично, что какой бы $x$ мы ни подставили, все равно получится «ноль равен нулю», т.е. верное числовое равенство.

А теперь давайте посмотрим, как всё это работает на примере реальных задач.

Примеры решения уравнений

Сегодня мы занимаемся линейными уравнениями, причем только простейшими. Вообще, под линейным уравнением подразумевается всякое равенство, содержащее в себе ровно одну переменную, и она идет лишь в первой степени.

Решаются такие конструкции примерно одинаково:

  1. Прежде всего необходимо раскрыть скобки, если они есть (как в нашем последнем примере);
  2. Затем свести подобные
  3. Наконец, уединить переменную, т.е. всё, что связано с переменной — слагаемые, в которых она содержится — перенести в одну сторону, а всё, что останется без неё, перенести в другую сторону.

Затем, как правило, нужно привести подобные с каждой стороны полученного равенства, а после этого останется лишь разделить на коэффициент при «иксе», и мы получим окончательный ответ.

В теории это выглядит красиво и просто, однако на практике даже опытные ученики старших классов могут допускать обидные ошибки в достаточно простых линейных уравнениях. Обычно ошибки допускаются либо при раскрытии скобок, либо при подсчёте «плюсов» и «минусов».

Кроме того, бывает так, что линейное уравнение вообще не имеет решений, или так, что решением является вся числовая прямая, т.е. любое число. Эти тонкости мы и разберем в сегодняшнем уроке. Но начнем мы, как вы уже поняли, с самых простых задач.

Схема решения простейших линейных уравнений

Для начала давайте я еще раз напишу всю схему решения простейших линейных уравнений:

  1. Раскрываем скобки, если они есть.
  2. Уединяем переменные, т.е. все, что содержит «иксы» переносим в одну сторону, а без «иксов» — в другую.
  3. Приводим подобные слагаемые.
  4. Разделяем все на коэффициент при «иксе».

Разумеется, эта схема работает не всегда, в ней есть определенные тонкости и хитрости, и сейчас мы с ними и познакомимся.

Решаем реальные примеры простых линейных уравнений

Задача №1

На первом шаге от нас требуется раскрыть скобки. Но их в этом примере нет, поэтому пропускаем данный этап. На втором шаге нам нужно уединить переменные. Обратите внимание: речь идет лишь об отдельных слагаемых. Давайте запишем:

Приводим подобные слагаемые слева и справа, но тут уже это сделано. Поэтому переходим к четвертому шагу: разделить на коэффициент:

\[\frac{6x}{6}=-\frac{72}{6}\]

Вот мы и получили ответ.

Задача №2

В этой задаче мы можем наблюдать скобки, поэтому давайте раскроем их:

И слева и справа мы видим примерно одну и ту же конструкцию, но давайте действовать по алгоритму, т.е. уединяем переменные:

Приведем подобные:

При каких корнях это выполняется. Ответ: при любых. Следовательно, можно записать, что $x$ — любое число.

Задача №3

Третье линейное уравнение уже интересней:

\[\left(6-x \right)+\left(12+x \right)-\left(3-2x \right)=15\]

Тут есть несколько скобок, однако они ни на что не умножаются, просто перед ними стоят различные знаки. Давайте раскроем их:

Выполняем второй уже известный нам шаг:

\[-x+x+2x=15-6-12+3\]

Посчитаем:

Выполняем последний шаг — делим все на коэффициент при «икс»:

\[\frac{2x}{x}=\frac{0}{2}\]

Что необходимо помнить при решении линейных уравнений

Если отвлечься от слишком простых задач, то я бы хотел сказать следующее:

  • Как я говорил выше, далеко не каждое линейное уравнение имеет решение — иногда корней просто нет;
  • Даже если корни есть, среди них может затесаться ноль — ничего страшного в этом нет.

Ноль — такое же число, как и остальные, не стоит его как-то дискриминировать или считать, что если у вас получился ноль, то вы что-то сделали неправильно.

Еще одна особенность связана с раскрытием скобок. Обратите внимание: когда перед ними стоит «минус», то мы его убираем, однако в скобках знаки меняем на противоположные . А дальше мы можем раскрывать ее по стандартным алгоритмам: мы получим то, что видели в выкладках выше.

Понимание этого простого факта позволит вам не допускать глупые и обидные ошибки в старших классах, когда выполнение подобных действий считается самим собой разумеющимся.

Решение сложных линейных уравнений

Перейдем к более сложным уравнениям. Теперь конструкции станут сложнее и при выполнении различных преобразований возникнет квадратичная функция. Однако не стоит этого бояться, потому что если по замыслу автора мы решаем линейное уравнение, то в процессе преобразования все одночлены, содержащие квадратичную функцию, обязательно сократятся.

Пример №1

Очевидно, что первым делом нужно раскрыть скобки. Давайте это сделаем очень аккуратно:

Теперь займемся уединением:

\[-x+6{{x}^{2}}-6{{x}^{2}}+x=-12\]

Приводим подобные:

Очевидно, что у данного уравнения решений нет, поэтому в ответе так и запишем:

\[\varnothing \]

или корней нет.

Пример №2

Выполняем те же действия. Первый шаг:

Перенесем все, что с переменной, влево, а без нее — вправо:

Приводим подобные:

Очевидно, что данное линейное уравнение не имеет решения, поэтому так и запишем:

\[\varnothing \],

либо корней нет.

Нюансы решения

Оба уравнения полностью решены. На примере этих двух выражений мы ещё раз убедились, что даже в самых простых линейных уравнениях всё может быть не так просто: корней может быть либо один, либо ни одного, либо бесконечно много. В нашем случае мы рассмотрели два уравнения, в обоих корней просто нет.

Но я бы хотел обратить ваше внимание на другой факт: как работать со скобками и как их раскрывать, если перед ними стоит знак «минус». Рассмотрим вот это выражение:

Прежде чем раскрывать, нужно перемножить всё на «икс». Обратите внимание: умножается каждое отдельное слагаемое . Внутри стоит два слагаемых — соответственно, два слагаемых и умножается.

И только после того, когда эти, казалось бы, элементарные, но очень важные и опасные преобразования выполнены, можно раскрывать скобку с точки зрения того, что после неё стоит знак «минус». Да, да: только сейчас, когда преобразования выполнены, мы вспоминаем, что перед скобками стоит знак «минус», а это значит, что все, что в низ, просто меняет знаки. При этом сами скобки исчезают и, что самое главное, передний «минус» тоже исчезает.

Точно также мы поступаем и со вторым уравнением:

Я не случайно обращаю внимание на эти мелкие, казалось бы, незначительные факты. Потому что решение уравнений — это всегда последовательность элементарных преобразований, где неумение чётко и грамотно выполнять простые действия приводит к тому, что ученики старших классов приходят ко мне и вновь учатся решать вот такие простейшие уравнения.

Разумеется, придёт день, и вы отточите эти навыки до автоматизма. Вам уже не придётся каждый раз выполнять столько преобразований, вы всё будете писать в одну строчку. Но пока вы только учитесь, нужно писать каждое действие отдельно.

Решение ещё более сложных линейных уравнений

То, что мы сейчас будем решать, уже сложно назвать простейшими задача, однако смысл остается тем же самым.

Задача №1

\[\left(7x+1 \right)\left(3x-1 \right)-21{{x}^{2}}=3\]

Давайте перемножим все элементы в первой части:

Давайте выполним уединение:

Приводим подобные:

Выполняем последний шаг:

\[\frac{-4x}{4}=\frac{4}{-4}\]

Вот наш окончательный ответ. И, несмотря на то, что у нас в процессе решения возникали коэффициенты с квадратичной функцией, однако они взаимно уничтожились, что делает уравнение именно линейным, а не квадратным.

Задача №2

\[\left(1-4x \right)\left(1-3x \right)=6x\left(2x-1 \right)\]

Давайте аккуратно выполним первый шаг: умножаем каждый элемент из первой скобки на каждый элемент из второй. Всего должно получиться четыре новых слагаемых после преобразований:

А теперь аккуратно выполним умножение в каждом слагаемом:

Перенесем слагаемые с «иксом» влево, а без — вправо:

\[-3x-4x+12{{x}^{2}}-12{{x}^{2}}+6x=-1\]

Приводим подобные слагаемые:

Мы вновь получили окончательный ответ.

Нюансы решения

Важнейшее замечание по поводу этих двух уравнений состоит в следующем: как только мы начинаем умножать скобки, в которых находится более чем оно слагаемое, то выполняется это по следующему правилу: мы берем первое слагаемое из первой и перемножаем с каждым элементом со второй; затем берем второй элемент из первой и аналогично перемножаем с каждым элементом со второй. В итоге у нас получится четыре слагаемых.

Об алгебраической сумме

На последнем примере я хотел бы напомнить ученикам, что такое алгебраическая сумма. В классической математике под $1-7$ мы подразумеваем простую конструкцию: из единицы вычитаем семь. В алгебре же мы подразумеваем под этим следующее: к числу «единица» мы прибавляем другое число, а именно «минус семь». Этим алгебраическая сумма отличается от обычной арифметической.

Как только при выполнении всех преобразований, каждого сложения и умножения вы начнёте видеть конструкции, аналогичные вышеописанным, никаких проблем в алгебре при работе с многочленами и уравнениями у вас просто не будет.

В заключение давайте рассмотрим ещё пару примеров, которые будут ещё более сложными, чем те, которые мы только что рассмотрели, и для их решения нам придётся несколько расширить наш стандартный алгоритм.

Решение уравнений с дробью

Для решения подобных заданий к нашему алгоритму придется добавить еще один шаг. Но для начала я напомню наш алгоритм:

  1. Раскрыть скобки.
  2. Уединить переменные.
  3. Привести подобные.
  4. Разделить на коэффициент.

Увы, этот прекрасный алгоритм при всей его эффективности оказывается не вполне уместным, когда перед нами дроби. А в том, что мы увидим ниже, у нас и слева, и справа в обоих уравнениях есть дробь.

Как работать в этом случае? Да всё очень просто! Для этого в алгоритм нужно добавить ещё один шаг, который можно совершить как перед первым действием, так и после него, а именно избавиться от дробей. Таким образом, алгоритм будет следующим:

  1. Избавиться от дробей.
  2. Раскрыть скобки.
  3. Уединить переменные.
  4. Привести подобные.
  5. Разделить на коэффициент.

Что значит «избавиться от дробей»? И почему выполнять это можно как после, так и перед первым стандартным шагом? На самом деле в нашем случае все дроби являются числовыми по знаменателю, т.е. везде в знаменателе стоит просто число. Следовательно, если мы обе части уравнения домножим на это число, то мы избавимся от дробей.

Пример №1

\[\frac{\left(2x+1 \right)\left(2x-3 \right)}{4}={{x}^{2}}-1\]

Давайте избавимся от дробей в этом уравнении:

\[\frac{\left(2x+1 \right)\left(2x-3 \right)\cdot 4}{4}=\left({{x}^{2}}-1 \right)\cdot 4\]

Обратите внимание: на «четыре» умножается все один раз, т.е. если у вас две скобки, это не значит, что каждую из них нужно умножать на «четыре». Запишем:

\[\left(2x+1 \right)\left(2x-3 \right)=\left({{x}^{2}}-1 \right)\cdot 4\]

Теперь раскроем:

Выполняем уединение переменной:

Выполняем приведение подобных слагаемых:

\[-4x=-1\left| :\left(-4 \right) \right.\]

\[\frac{-4x}{-4}=\frac{-1}{-4}\]

Мы получили окончательное решение, переходим ко второму уравнению.

Пример №2

\[\frac{\left(1-x \right)\left(1+5x \right)}{5}+{{x}^{2}}=1\]

Здесь выполняем все те же действия:

\[\frac{\left(1-x \right)\left(1+5x \right)\cdot 5}{5}+{{x}^{2}}\cdot 5=5\]

\[\frac{4x}{4}=\frac{4}{4}\]

Задача решена.

Вот, собственно, и всё, что я хотел сегодня рассказать.

Ключевые моменты

Ключевые выводы следующие:

  • Знать алгоритм решения линейных уравнений.
  • Умение раскрывать скобки.
  • Не стоит переживать, если где-то у вас появляются квадратичные функции, скорее всего, в процессе дальнейших преобразований они сократятся.
  • Корни в линейных уравнениях, даже самых простых, бывают трех типов: один единственный корень, вся числовая прямая является корнем, корней нет вообще.

Надеюсь, этот урок поможет вам освоить несложную, но очень важную для дальнейшего понимания всей математики тему. Если что-то непонятно, заходите на сайт, решайте примеры, представленные там. Оставайтесь с нами, вас ждет еще много интересного!

Уравнения — одна из сложных тем для усвоения, но при этом они являются достаточно мощным инструментом для решения большинства задач.

С помощью уравнений описываются различные процессы, протекающие в природе. Уравнения широко применяются в других науках: в экономике, физике, биологии и химии.

В данном уроке мы попробуем понять суть простейших уравнений, научимся выражать неизвестные и решим несколько уравнений. По мере усвоения новых материалов, уравнения будут усложняться, поэтому понять основы очень важно.

Предварительные навыки Содержание урока

Что такое уравнение?

Уравнение — это равенство, содержащее в себе переменную, значение которой требуется найти. Это значение должно быть таким, чтобы при его подстановке в исходное уравнение получалось верное числовое равенство.

Например выражение 2 + 2 = 4 является равенством. При вычислении левой части получается верное числовое равенство 4 = 4 .

А вот равенство 2 + x = 4 является уравнением, поскольку содержит в себе переменную x , значение которой можно найти. Значение должно быть таким, чтобы при подстановке этого значения в исходное уравнение, получилось верное числовое равенство.

Другими словами, мы должны найти такое значение, при котором знак равенства оправдал бы свое местоположение — левая часть должна быть равна правой части.

Уравнение 2 + x = 4 является элементарным. Значение переменной x равно числу 2. При любом другом значении равенство соблюдаться не будет

Говорят, что число 2 является корнем или решением уравнения 2 + x = 4

Корень или решение уравнения — это значение переменной, при котором уравнение обращается в верное числовое равенство.

Корней может быть несколько или не быть совсем. Решить уравнение означает найти его корни или доказать, что корней нет.

Переменную, входящую в уравнение, иначе называют неизвестным . Вы вправе называть как вам удобнее. Это синонимы.

Примечание . Словосочетание «решить уравнение» говорит само за себя. Решить уравнение означает «уравнять» равенство — сделать его сбалансированным, чтобы левая часть равнялась правой части.

Выразить одно через другое

Изучение уравнений по традиции начинается с того, чтобы научиться выражать одно число, входящее в равенство, через ряд других. Давайте не будем нарушать эту традицию и поступим также.

Рассмотрим следующее выражение:

8 + 2

Данное выражение является суммой чисел 8 и 2. Значение данного выражения равно 10

8 + 2 = 10

Получили равенство. Теперь можно выразить любое число из этого равенства через другие числа, входящие в это же равенство. К примеру, выразим число 2.

Чтобы выразить число 2, нужно задать вопрос: «что нужно сделать с числами 10 и 8, чтобы получить число 2». Понятно, что для получения числа 2, нужно из числа 10 вычесть число 8.

Так и делаем. Записываем число 2 и через знак равенства говорим, что для получения этого числа 2 мы из числа 10 вычли число 8:

2 = 10 − 8

Мы выразили число 2 из равенства 8 + 2 = 10 . Как видно из примера, ничего сложного в этом нет.

При решении уравнений, в частности при выражении одного числа через другие, знак равенства удобно заменять на слово «есть» . Делать это нужно мысленно, а не в самом выражении.

Так, выражая число 2 из равенства 8 + 2 = 10 мы получили равенство 2 = 10 − 8 . Данное равенство можно прочесть так:

2 есть 10 − 8

То есть, знак = заменен на слово «есть». Более того, равенство 2 = 10 − 8 можно перевести с математического языка на полноценный человеческий язык. Тогда егоможно будет прочитать следующим образом:

Число 2 есть разность числа 10 и числа 8

Число 2 есть разница между числом 10 и числом 8.

Но мы ограничимся лишь заменой знака равенства на слово «есть», и то будем делать это не всегда. Элементарные выражения можно понимать и без перевода математического языка на язык человеческий.

Вернём получившееся равенство 2 = 10 − 8 в первоначальное состояние:

8 + 2 = 10

Выразим в этот раз число 8. Что нужно сделать с остальными числами, чтобы получить число 8? Верно, нужно из числа 10 вычесть число 2

8 = 10 − 2

Вернем получившееся равенство 8 = 10 − 2 в первоначальное состояние:

8 + 2 = 10

В этот раз выразим число 10. Но оказывается, что десятку выражать не нужно, поскольку она уже выражена. Достаточно поменять местами левую и правую часть, тогда получится то, что нам нужно:

10 = 8 + 2

Пример 2 . Рассмотрим равенство 8 − 2 = 6

Выразим из этого равенства число 8. Чтобы выразить число 8 остальные два числа нужно сложить:

8 = 6 + 2

Вернем получившееся равенство 8 = 6 + 2 в первоначальное состояние:

8 − 2 = 6

Выразим из этого равенства число 2. Чтобы выразить число 2, нужно из 8 вычесть 6

2 = 8 − 6

Пример 3 . Рассмотрим равенство 3 × 2 = 6

Выразим число 3. Чтобы выразить число 3, нужно 6 разделить 2

Вернем получившееся равенство в первоначальное состояние:

3 × 2 = 6

Выразим из этого равенства число 2. Чтобы выразить число 2, нужно 6 разделить 3

Пример 4 . Рассмотрим равенство

Выразим из этого равенства число 15. Чтобы выразить число 15, нужно перемножить числа 3 и 5

15 = 3 × 5

Вернем получившееся равенство 15 = 3 × 5 в первоначальное состояние:

Выразим из этого равенства число 5. Чтобы выразить число 5, нужно 15 разделить 3

Правила нахождения неизвестных

Рассмотрим несколько правил нахождения неизвестных. Возможно, они вам знакомы, но не мешает повторить их ещё раз. В дальнейшем их можно будет забыть, поскольку мы научимся решать уравнения, не применяя эти правила.

Вернемся к первому примеру, который мы рассматривали в предыдущей теме, где в равенстве 8 + 2 = 10 требовалось выразить число 2.

В равенстве 8 + 2 = 10 числа 8 и 2 являются слагаемыми, а число 10 — суммой.

Чтобы выразить число 2, мы поступили следующим образом:

2 = 10 − 8

То есть, из суммы 10 вычли слагаемое 8.

Теперь представим, что в равенстве 8 + 2 = 10 вместо числа 2 располагается переменная x

8 + x = 10

В этом случае равенство 8 + 2 = 10 превращается в уравнение 8 + x = 10 , а переменная x неизвестного слагаемого

Наша задача найти это неизвестное слагаемое, то есть решить уравнение 8 + x = 10 . Для нахождения неизвестного слагаемого предусмотрено следующее правило:

Чтобы найти неизвестное слагаемое, нужно из суммы вычесть известное слагаемое.

Что мы в принципе и сделали, когда выражали двойку в равенстве 8 + 2 = 10 . Чтобы выразить слагаемое 2, мы из суммы 10 вычли другое слагаемое 8

2 = 10 − 8

А сейчас, чтобы найти неизвестное слагаемое x , мы должны из суммы 10 вычесть известное слагаемое 8:

x = 10 − 8

Если вычислить правую часть получившегося равенства, то можно узнать чему равна переменная x

x = 2

Мы решили уравнение. Значение переменной x равно 2 . Для проверки значение переменной x отправляют в исходное уравнение 8 + x = 10 и подставляют вместо x. Так желательно поступать с любым решённым уравнением, поскольку нельзя быть точно уверенным, что уравнение решено правильно:

В результате

Это же правило действовало бы в случае, если неизвестным слагаемым было бы первое число 8.

x + 2 = 10

В этом уравнении x — это неизвестное слагаемое, 2 — известное слагаемое, 10 — сумма. Чтобы найти неизвестное слагаемое x , нужно из суммы 10 вычесть известное слагаемое 2

x = 10 − 2

x = 8

Вернемся ко второму примеру из предыдущей темы, где в равенстве 8 − 2 = 6 требовалось выразить число 8.

В равенстве 8 − 2 = 6 число 8 это уменьшаемое, число 2 — вычитаемое, число 6 — разность

Чтобы выразить число 8, мы поступили следующим образом:

8 = 6 + 2

То есть, сложили разность 6 и вычитаемое 2.

Теперь представим, что в равенстве 8 − 2 = 6 вместо числа 8 располагается переменная x

x − 2 = 6

В этом случае переменная x берет на себя роль так называемого неизвестного уменьшаемого

Для нахождения неизвестного уменьшаемого предусмотрено следующее правило:

Чтобы найти неизвестное уменьшаемое, нужно к разности прибавить вычитаемое.

Что мы и сделали, когда выражали число 8 в равенстве 8 − 2 = 6 . Чтобы выразить уменьшаемое 8, мы к разности 6 прибавили вычитаемое 2.

А сейчас, чтобы найти неизвестное уменьшаемое x , мы должны к разности 6 прибавить вычитаемое 2

x = 6 + 2

Если вычислить правую часть, то можно узнать чему равна переменная x

x = 8

Теперь представим, что в равенстве 8 − 2 = 6 вместо числа 2 располагается переменная x

8 − x = 6

В этом случае переменная x берет на себя роль неизвестного вычитаемого

Для нахождения неизвестного вычитаемого предусмотрено следующее правило:

Чтобы найти неизвестное вычитаемое, нужно из уменьшаемого вычесть разность.

Что мы и сделали, когда выражали число 2 в равенстве 8 − 2 = 6. Чтобы выразить число 2, мы из уменьшаемого 8 вычли разность 6.

А сейчас, чтобы найти неизвестное вычитаемое x , нужно опять же из уменьшаемого 8 вычесть разность 6

x = 8 − 6

Вычисляем правую часть и находим значение x

x = 2

Вернемся к третьему примеру из предыдущей темы, где в равенстве 3 × 2 = 6 мы пробовали выразить число 3.

В равенстве 3 × 2 = 6 число 3 — это множимое, число 2 — множитель, число 6 — произведение

Чтобы выразить число 3 мы поступили следующим образом:

То есть, разделили произведение 6 на множитель 2.

Теперь представим, что в равенстве 3 × 2 = 6 вместо числа 3 располагается переменная x

x × 2 = 6

В этом случае переменная x берет на себя роль неизвестного множимого .

Для нахождения неизвестного множимого предусмотрено следующее правило:

Чтобы найти неизвестное множимое, нужно произведение разделить на множитель.

Что мы и сделали, когда выражали число 3 из равенства 3 × 2 = 6 . Произведение 6 мы разделили на множитель 2.

А сейчас для нахождения неизвестного множимого x , нужно произведение 6 разделить на множитель 2.

Вычисление правой части позволяет нам найти значение переменной x

x = 3

Это же правило применимо в случае, если переменная x располагается вместо множителя, а не множимого. Представим, что в равенстве 3 × 2 = 6 вместо числа 2 располагается переменная x .

В этом случае переменная x берет на себя роль неизвестного множителя . Для нахождения неизвестного множителя предусмотрено такое же, что и для нахождения неизвестного множимого, а именно деление произведения на известный множитель:

Чтобы найти неизвестный множитель, нужно произведение разделить на множимое.

Что мы и сделали, когда выражали число 2 из равенства 3 × 2 = 6 . Тогда для получения числа 2 мы разделили произведение 6 на множимое 3.

А сейчас для нахождения неизвестного множителя x мы разделили произведение 6 на множимое 3.

Вычисление правой части равенства позволяет узнать чему равно x

x = 2

Множимое и множитель вместе называют сомножителями. Поскольку правила нахождения множимого и множителя совпадают, мы можем сформулировать общее правило нахождения неизвестного сомножителя:

Чтобы найти неизвестный сомножитель, нужно произведение разделить на известный сомножитель.

Например, решим уравнение 9 × x = 18 . Переменная x является неизвестным сомножителем. Чтобы найти этот неизвестный сомножитель, нужно произведение 18 разделить на известный сомножитель 9

Решим уравнение x × 3 = 27 . Переменная x является неизвестным сомножителем. Чтобы найти этот неизвестный сомножитель, нужно произведение 27 разделить на известный сомножитель 3

Вернемся к четвертому примеру из предыдущей темы, где в равенстве требовалось выразить число 15. В этом равенстве число 15 — это делимое, число 5 — делитель, число 3 — частное.

Чтобы выразить число 15 мы поступили следующим образом:

15 = 3 × 5

То есть, умножили частное 3 на делитель 5.

Теперь представим, что в равенстве вместо числа 15 располагается переменная x

В этом случае переменная x берет на себя роль неизвестного делимого .

Для нахождения неизвестного делимого предусмотрено следующее правило:

Чтобы найти неизвестное делимое, нужно частное умножить на делитель.

Что мы и сделали, когда выражали число 15 из равенства . Чтобы выразить число 15, мы умножили частное 3 на делитель 5.

А сейчас, чтобы найти неизвестное делимое x , нужно частное 3 умножить на делитель 5

x = 3 × 5

x .

x = 15

Теперь представим, что в равенстве вместо числа 5 располагается переменная x .

В этом случае переменная x берет на себя роль неизвестного делителя .

Для нахождения неизвестного делителя предусмотрено следующее правило:

Что мы и сделали, когда выражали число 5 из равенства . Чтобы выразить число 5, мы разделили делимое 15 на частное 3.

А сейчас, чтобы найти неизвестный делитель x , нужно делимое 15 разделить на частное 3

Вычислим правую часть получившегося равенства. Так мы узнаем чему равна переменная x .

x = 5

Итак, для нахождения неизвестных мы изучили следующие правила:

  • Чтобы найти неизвестное слагаемое, нужно из суммы вычесть известное слагаемое;
  • Чтобы найти неизвестное уменьшаемое, нужно к разности прибавить вычитаемое;
  • Чтобы найти неизвестное вычитаемое, нужно из уменьшаемого вычесть разность;
  • Чтобы найти неизвестное множимое, нужно произведение разделить на множитель;
  • Чтобы найти неизвестный множитель, нужно произведение разделить на множимое;
  • Чтобы найти неизвестное делимое, нужно частное умножить на делитель;
  • Чтобы найти неизвестный делитель, нужно делимое разделить на частное.

Компоненты

Компонентами мы будем называть числа и переменные, входящие в равенство

Так, компонентами сложения являются слагаемые и сумма

Компонентами вычитания являются уменьшаемое , вычитаемое и разность

Компонентами умножения являются множимое , множитель и произведение

Компонентами деления являются делимое, делитель и частное

В зависимости от того, с какими компонентами мы будем иметь дело, будут применяться соответствующие правила нахождения неизвестных. Эти правила мы изучили в предыдущей теме. При решении уравнений желательно знать эти правило наизусть.

Пример 1 . Найти корень уравнения 45 + x = 60

45 — слагаемое, x — неизвестное слагаемое, 60 — сумма. Имеем дело с компонентами сложения. Вспоминаем, что для нахождения неизвестного слагаемого, нужно из суммы вычесть известное слагаемое:

x = 60 − 45

Вычислим правую часть, получим значение x равное 15

x = 15

Значит корень уравнения 45 + x = 60 равен 15.

Чаще всего неизвестное слагаемое необходимо привести к виду при котором его можно было бы выразить.

Пример 2 . Решить уравнение

Здесь в отличие от предыдущего примера, неизвестное слагаемое нельзя выразить сразу, поскольку оно содержит коэффициент 2. Наша задача привести это уравнение к виду при котором можно было бы выразить x

В данном примере мы имеем дело с компонентами сложения — слагаемыми и суммой. 2x — это первое слагаемое, 4 — второе слагаемое, 8 — сумма.

При этом слагаемое 2x содержит переменную x . После нахождения значения переменной x слагаемое 2x примет другой вид. Поэтому слагаемое 2x можно полностью принять за неизвестное слагаемое:

Теперь применяем правило нахождения неизвестного слагаемого. Вычитаем из суммы известное слагаемое:

Вычислим правую часть получившегося уравнения:

Мы получили новое уравнение . Теперь мы имеем дело с компонентами умножения: множимым, множителем и произведением. 2 — множимое, x — множитель, 4 — произведение

При этом переменная x является не просто множителем, а неизвестным множителем

Чтобы найти этот неизвестный множитель, нужно произведение разделить на множимое:

Вычислим правую часть, получим значение переменной x

Для проверки найденный корень отправим в исходное уравнение и подставим вместо x

Пример 3 . Решить уравнение 3x + 9x + 16x = 56

Cразу выразить неизвестное x нельзя. Сначала нужно привести данное уравнение к виду при котором его можно было бы выразить.

Приведем в левой части данного уравнения:

Имеем дело с компонентами умножения. 28 — множимое, x — множитель, 56 — произведение. При этом x является неизвестным множителем. Чтобы найти неизвестный множитель, нужно произведение разделить на множимое:

Отсюда x равен 2

Равносильные уравнения

В предыдущем примере при решении уравнения 3x + 9x + 16x = 56 , мы привели подобные слагаемые в левой части уравнения. В результате получили новое уравнение 28x = 56 . Старое уравнение 3x + 9x + 16x = 56 и получившееся новое уравнение 28x = 56 называют равносильными уравнениями , поскольку их корни совпадают.

Уравнения называют равносильными, если их корни совпадают.

Проверим это. Для уравнения 3x + 9x + 16x = 56 мы нашли корень равный 2 . Подставим этот корень сначала в уравнение 3x + 9x + 16x = 56 , а затем в уравнение 28x = 56 , которое получилось в результате приведения подобных слагаемых в левой части предыдущего уравнения. Мы должны получить верные числовые равенства

Согласно порядку действий, в первую очередь выполняется умножение:

Подставим корень 2 во второе уравнение 28x = 56

Видим, что у обоих уравнений корни совпадают. Значит уравнения 3x + 9x + 16x = 6 и 28x = 56 действительно являются равносильными.

Для решения уравнения 3x + 9x + 16x = 56 мы воспользовались одним из — приведением подобных слагаемых. Правильное тождественное преобразование уравнения позволило нам получить равносильное уравнение 28x = 56 , которое проще решать.

Из тождественных преобразований на данный момент мы умеем только сокращать дроби, приводить подобные слагаемые, выносить общий множитель за скобки, а также раскрывать скобки. Существуют и другие преобразования, которые следует знать. Но для общего представления о тождественных преобразованиях уравнений, изученных нами тем вполне хватает.

Рассмотрим некоторые преобразования, которые позволяют получить равносильное уравнение

Если к обеим частям уравнения прибавить одно и то же число, то получится уравнение равносильное данному.

и аналогично:

Если из обеих частей уравнения вычесть одно и то же число, то получится уравнение равносильное данному.

Другими словами, корень уравнения не изменится, если к обеим частям данного уравнения прибавить (или вычесть из обеих частей) одно и то же число.

Пример 1 . Решить уравнение

Вычтем из обеих частей уравнения число 10

Получили уравнение 5x = 10 . Имеем дело с компонентами умножения. Чтобы найти неизвестный сомножитель x , нужно произведение 10 разделить на известный сомножитель 5.

и подставим вместо x найденное значение 2

Получили верное числовое равенство. Значит уравнение решено правильно.

Решая уравнение мы вычли из обеих частей уравнения число 10 . В результате получили равносильное уравнение . Корень этого уравнения, как и уравнения так же равен 2

Пример 2 . Решить уравнение 4(x + 3) = 16

Вычтем из обеих частей уравнения число 12

В левой части останется 4x , а в правой части число 4

Получили уравнение 4x = 4 . Имеем дело с компонентами умножения. Чтобы найти неизвестный сомножитель x , нужно произведение 4 разделить на известный сомножитель 4

Вернемся к исходному уравнению 4(x + 3) = 16 и подставим вместо x найденное значение 1

Получили верное числовое равенство. Значит уравнение решено правильно.

Решая уравнение 4(x + 3) = 16 мы вычли из обеих частей уравнения число 12 . В результате получили равносильное уравнение 4x = 4 . Корень этого уравнения, как и уравнения 4(x + 3) = 16 так же равен 1

Пример 3 . Решить уравнение

Раскроем скобки в левой части равенства:

Прибавим к обеим частям уравнения число 8

Приведем подобные слагаемые в обеих частях уравнения:

В левой части останется 2x , а в правой части число 9

В получившемся уравнении 2x = 9 выразим неизвестное слагаемое x

Вернемся к исходному уравнению и подставим вместо x найденное значение 4,5

Получили верное числовое равенство. Значит уравнение решено правильно.

Решая уравнение мы прибавили к обеим частям уравнения число 8. В результате получили равносильное уравнение . Корень этого уравнения, как и уравнения так же равен 4,5

Следующее правило, которое позволяет получить равносильное уравнение, выглядит следующим образом

Если в уравнении перенести слагаемое из одной части в другую, изменив его знак, то получится уравнение равносильное данному.

То есть, корень уравнения не изменится, если мы перенесем слагаемое из одной части уравнения в другую, изменив его знак. Это свойство является одним из важных и одним из часто используемых при решении уравнений.

Рассмотрим следующее уравнение:

Корень данного уравнения равен 2. Подставим вместо x этот корень и проверим получается ли верное числовое равенство

Получается верное равенство. Значит число 2 действительно является корнем уравнения .

Теперь попробуем поэкспериментировать со слагаемыми этого уравнения, перенося их из одной части в другую, изменяя знаки.

Например, слагаемое 3x располагается в левой части равенства. Перенесём его в правую часть, изменив знак на противоположный:

Получилось уравнение 12 = 9x − 3x . в правой части данного уравнения:

x является неизвестным сомножителем. Найдём этот известный сомножитель:

Отсюда x = 2 . Как видим, корень уравнения не изменился. Значит уравнения 12 + 3x = 9x и 12 = 9x − 3x являются равносильными.

На самом деле, данное преобразование является упрощенным методом предыдущего преобразования, где к обеим частях уравнения прибавлялось (или вычиталось) одно и то же число.

Мы сказали, что в уравнении 12 + 3x = 9x слагаемое 3x было перенесено в правую часть, изменив знак. В реальности же происходило следующее: из обеих частей уравнения вычли слагаемое 3x

Затем в левой части были приведены подобные слагаемые и получено уравнение 12 = 9x − 3x. Затем опять были приведены подобные слагаемые, но уже в правой части, и получено уравнение 12 = 6x.

Но так называемый «перенос» более удобен для подобных уравнений, поэтому он и получил такое широкое распространение. Решая уравнения, мы часто будем пользоваться именно этим преобразованием.

Равносильными также являются уравнения 12 + 3x = 9x и 3x − 9x = −12 . В этот раз в уравнении 12 + 3x = 9x слагаемое 12 было перенесено в правую часть, а слагаемое 9x в левую. Не следует забывать, что знаки этих слагаемых были изменены во время переноса

Следующее правило, которое позволяет получить равносильное уравнение, выглядит следующим образом:

Если обе части уравнения умножить или разделить на одно и то же число, не равное нулю, то получится уравнение равносильное данному.

Другими словами, корни уравнения не изменятся, если обе его части умножить или разделить на одно и то же число. Это действие часто применяется тогда, когда нужно решить уравнение содержащее дробные выражения.

Сначала рассмотрим примеры, в которых обе части уравнения будут умножаться на одно и то же число.

Пример 1 . Решить уравнение

При решении уравнений, содержащих дробные выражения, сначала принято упростить это уравнение.

В данном случае мы имеем дело именно с таким уравнением. В целях упрощения данного уравнения обе его части можно умножить на 8:

Мы помним, что для , нужно числитель данной дроби умножить на это число. У нас имеются две дроби и каждая из них умножается на число 8. Наша задача умножить числители дробей на это число 8

Теперь происходит самое интересное. В числителях и знаменателях обеих дробей содержится множитель 8, который можно сократить на 8. Это позволит нам избавиться от дробного выражения:

В результате останется простейшее уравнение

Ну и нетрудно догадаться, что корень этого уравнения равен 4

x найденное значение 4

Получается верное числовое равенство. Значит уравнение решено правильно.

При решении данного уравнения мы умножили обе его части на 8. В результате получили уравнение . Корень этого уравнения, как и уравнения равен 4. Значит эти уравнения равносильны.

Множитель на который умножаются обе части уравнения принято записывать перед частью уравнения, а не после неё. Так, решая уравнение , мы умножили обе части на множитель 8 и получили следующую запись:

От этого корень уравнения не изменился, но если бы мы сделали это находясь в школе, то нам сделали бы замечание, поскольку в алгебре множитель принято записывать перед тем выражением, с которым он перемножается. Поэтому умножение обеих частей уравнения на множитель 8 желательно переписать следующим образом:

Пример 2 . Решить уравнение

В левой части множители 15 можно сократить на 15, а в правой части множители 15 и 5 можно сократить на 5

Раскроем скобки в правой части уравнения:

Перенесем слагаемое x из левой части уравнения в правую часть, изменив знак. А слагаемое 15 из правой части уравнения перенесем в левую часть, опять же изменив знак:

Приведем подобные слагаемые в обеих частях, получим

Имеем дело с компонентами умножения. Переменная x

Вернемся к исходному уравнению и подставим вместо x найденное значение 5

Получается верное числовое равенство. Значит уравнение решено правильно. При решении данного уравнения мы умножили обе го части на 15 . Далее выполняя тождественные преобразования, мы получили уравнение 10 = 2x . Корень этого уравнения, как и уравнения равен 5 . Значит эти уравнения равносильны.

Пример 3 . Решить уравнение

В левой части можно сократить две тройки, а правая часть будет равна 18

Останется простейшее уравнение . Имеем дело с компонентами умножения. Переменная x является неизвестным сомножителем. Найдём этот известный сомножитель:

Вернемся к исходному уравнению и подставим вместо x найденное значение 9

Получается верное числовое равенство. Значит уравнение решено правильно.

Пример 4 . Решить уравнение

Умножим обе части уравнения на 6

В левой части уравнения раскроем скобки. В правой части множитель 6 можно поднять в числитель:

Сократим в обеих частях уравнениях то, что можно сократить:

Перепишем то, что у нас осталось:

Воспользуемся переносом слагаемых. Слагаемые, содержащие неизвестное x , сгруппируем в левой части уравнения, а слагаемые свободные от неизвестных — в правой:

Приведем подобные слагаемые в обеих частях:

Теперь найдем значение переменной x . Для этого разделим произведение 28 на известный сомножитель 7

Отсюда x = 4.

Вернемся к исходному уравнению и подставим вместо x найденное значение 4

Получилось верное числовое равенство. Значит уравнение решено правильно.

Пример 5 . Решить уравнение

Раскроем скобки в обеих частях уравнения там, где это можно:

Умножим обе части уравнения на 15

Раскроем скобки в обеих частях уравнения:

Сократим в обеих частях уравнения, то что можно сократить:

Перепишем то, что у нас осталось:

Раскроем скобки там, где это можно:

Воспользуемся переносом слагаемых. Слагаемые, содержащие неизвестное, сгруппируем в левой части уравнения, а слагаемые, свободные от неизвестных — в правой. Не забываем, что во время переноса, слагаемые меняют свои знаки на противоположные:

Приведем подобные слагаемые в обеих частях уравнения:

Найдём значение x

В получившемся ответе можно выделить целую часть:

Вернемся к исходному уравнению и подставим вместо x найденное значение

Получается довольно громоздкое выражение. Воспользуемся переменными. Левую часть равенства занесем в переменную A , а правую часть равенства в переменную B

Наша задача состоит в том, чтобы убедиться равна ли левая часть правой. Другими словами, доказать равенство A = B

Найдем значение выражения, находящегося в переменной А.

Значение переменной А равно . Теперь найдем значение переменной B . То есть, значение правой части нашего равенства. Если и оно равно , то уравнение будет решено верно

Видим, что значение переменной B , как и значение переменной A равно . Это значит, что левая часть равна правой части. Отсюда делаем вывод, что уравнение решено правильно.

Теперь попробуем не умножать обе части уравнения на одно и то же число, а делить.

Рассмотрим уравнение 30x + 14x + 14 = 70x − 40x + 42 . Решим его обычным методом: слагаемые, содержащие неизвестные, сгруппируем в левой части уравнения, а слагаемые, свободные от неизвестных — в правой. Далее выполняя известные тождественные преобразования, найдем значение x

Подставим найденное значение 2 вместо x в исходное уравнение:

Теперь попробуем разделить все слагаемые уравнения 30x + 14x + 14 = 70x − 40x + 42 на какое-нибудь число.Замечаем, что все слагаемые этого уравнения имеют общий множитель 2. На него и разделим каждое слагаемое:

Выполним сокращение в каждом слагаемом:

Перепишем то, что у нас осталось:

Решим это уравнение, пользуясь известными тождественными преобразованиями:

Получили корень 2 . Значит уравнения 15x + 7x + 7 = 35x − 20x + 21 и 30x + 14x + 14 = 70x − 40x + 42 равносильны.

Деление обеих частей уравнения на одно и то же число позволяет освобождать неизвестное от коэффициента. В предыдущем примере когда мы получили уравнение 7x = 14 , нам потребовалось разделить произведение 14 на известный сомножитель 7. Но если бы мы в левой части освободили неизвестное от коэффициента 7, корень нашелся бы сразу. Для этого достаточно было разделить обе части на 7

Этим методом мы тоже будем пользоваться часто.

Умножение на минус единицу

Если обе части уравнения умножить на минус единицу, то получится уравнение равносильное данному.

Это правило следует из того, что от умножения (или деления) обеих частей уравнения на одно и то же число, корень данного уравнения не меняется. А значит корень не поменяется если обе его части умножить на −1 .

Данное правило позволяет поменять знаки всех компонентов, входящих в уравнение. Для чего это нужно? Опять же, чтобы получить равносильное уравнение, которое проще решать.

Рассмотрим уравнение . Чему равен корень этого уравнения?

Прибавим к обеим частях уравнения число 5

Приведем подобные слагаемые:

А теперь вспомним про . Что же представляет собой левая часть уравнения . Это есть произведение минус единицы и переменной x

То есть, минус стоящий перед переменной x относится не к самой переменной x , а к единице, которую мы не видим, поскольку коэффициент 1 принято не записывать. Это означает, что уравнение на самом деле выглядит следующим образом:

Имеем дело с компонентами умножения. Чтобы найти х , нужно произведение −5 разделить на известный сомножитель −1 .

или разделить обе части уравнения на −1 , что еще проще

Итак, корень уравнения равен 5 . Для проверки подставим его в исходное уравнение. Не забываем, что в исходном уравнении минус стоящий перед переменной x относится к невидимой единице

Получилось верное числовое равенство. Значит уравнение решено верно.

Теперь попробуем умножить обе части уравнения на минус единицу:

После раскрытия скобок в левой части образуется выражение , а правая часть будет равна 10

Корень этого уравнения, как и уравнения равен 5

Значит уравнения и равносильны.

Пример 2 . Решить уравнение

В данном уравнении все компоненты являются отрицательными. С положительными компонентами работать удобнее, чем с отрицательными, поэтому поменяем знаки всех компонентов, входящих в уравнение . Для этого умножим обе части данного уравнения на −1 .

Понятно, что от умножения на −1 любое число поменяет свой знак на противоположный. Поэтому саму процедуру умножения на −1 и раскрытие скобок подробно не расписывают, а сразу записывают компоненты уравнения с противоположными знаками.

Так, умножение уравнения на −1 можно записать подробно следующим образом:

либо можно просто поменять знаки всех компонентов:

Получится то же самое, но разница будет в том, что мы сэкономим себе время.

Итак, умножив обе части уравнения на −1 , мы получили уравнение . Решим данное уравнение. Из обеих частей вычтем число 4 и разделим обе части на 3

Когда корень найден, переменную обычно записывают в левой части, а её значение в правой, что мы и сделали.

Пример 3 . Решить уравнение

Умножим обе части уравнения на −1 . Тогда все компоненты поменяют свои знаки на противоположные:

Из обеих частей получившегося уравнения вычтем 2x и приведем подобные слагаемые:

Прибавим к обеим частям уравнения единицу и приведем подобные слагаемые:

Приравнивание к нулю

Недавно мы узнали, что если в уравнении перенести слагаемое из одной части в другую, изменив его знак, то получится уравнение равносильное данному.

А что будет если перенести из одной части в другую не одно слагаемое, а все слагаемые? Верно, в той части откуда забрали все слагаемые останется ноль. Иными словами, не останется ничего.

В качестве примера рассмотрим уравнение . Решим данное уравнение, как обычно — слагаемые, содержащие неизвестные сгруппируем в одной части, а числовые слагаемые, свободные от неизвестных оставим в другой. Далее выполняя известные тождественные преобразования, найдем значение переменной x

Теперь попробуем решить это же уравнение, приравняв все его компоненты к нулю. Для этого перенесем все слагаемые из правой части в левую, изменив знаки:

Приведем подобные слагаемые в левой части:

Прибавим к обеим частям 77 , и разделим обе части на 7

Альтернатива правилам нахождения неизвестных

Очевидно, что зная о тождественных преобразованиях уравнений, можно не заучивать наизусть правила нахождения неизвестных.

К примеру, для нахождения неизвестного в уравнении мы произведение 10 делили на известный сомножитель 2

Но если в уравнении обе части разделить на 2 корень найдется сразу. В левой части уравнения в числителе множитель 2 и в знаменателе множитель 2 сократятся на 2. А правая часть будет равна 5

Уравнения вида мы решали выражая неизвестное слагаемое:

Но можно воспользоваться тождественными преобразованиями, которые мы сегодня изучили. В уравнении слагаемое 4 можно перенести в правую часть, изменив знак:

В левой части уравнения сократятся две двойки. Правая часть будет равна 2. Отсюда .

Либо можно было из обеих частей уравнения вычесть 4. Тогда получилось бы следующее:

В случае с уравнениями вида удобнее делить произведение на известный сомножитель. Сравним оба решения:

Первое решение намного короче и аккуратнее. Второе решение можно значительно укоротить, если выполнить деление в уме.

Тем не менее, необходимо знать оба метода, и только затем использовать тот, который больше нравится.

Когда корней несколько

Уравнение может иметь несколько корней. Например уравнение x (x + 9) = 0 имеет два корня: 0 и −9 .

В уравнении x (x + 9) = 0 нужно было найти такое значение x при котором левая часть была бы равна нулю. В левой части этого уравнения содержатся выражения x и (x + 9) , которые являются сомножителями. Из законов произведения мы знаем, что произведение равно нулю, если хотя бы один из сомножителей равен нулю (или первый сомножитель или второй).

То есть, в уравнении x (x + 9) = 0 равенство будет достигаться, если x будет равен нулю или (x + 9) будет равно нулю.

x = 0 или x + 9 = 0

Приравняв к нулю оба этих выражения, мы сможем найти корни уравнения x (x + 9) = 0 . Первый корень, как видно из примера, нашелся сразу. Для нахождения второго корня нужно решить элементарное уравнение x + 9 = 0 . Несложно догадаться, что корень этого уравнения равен −9 . Проверка показывает, что корень верный:

−9 + 9 = 0

Пример 2 . Решить уравнение

Данное уравнение имеет два корня: 1 и 2. Левая часть уравнения является произведение выражений (x − 1) и (x − 2) . А произведение равно нулю, если хотя бы один из сомножителей равен нулю (или сомножитель (x − 1) или сомножитель (x − 2) ).

Найдем такое x при котором выражения (x − 1) или (x − 2) обращаются в нули:

Подставляем по-очереди найденные значения в исходное уравнение и убеждаемся, что при этих значениях левая часть равняется нулю:

Когда корней бесконечно много

Уравнение может иметь бесконечно много корней. То есть, подставив в такое уравнение любое число, мы получим верное числовое равенство.

Пример 1 . Решить уравнение

Корнем данного уравнения является любое число. Если раскрыть скобки в левой части уравнения и привести подобные слагаемые, то получится равенство 14 = 14 . Это равенство будет получаться при любом x

Пример 2 . Решить уравнение

Корнем данного уравнения является любое число. Если раскрыть скобки в левой части уравнения, то получится равенство 10x + 12 = 10x + 12. Это равенство будет получаться при любом x

Когда корней нет

Случается и так, что уравнение вовсе не имеет решений, то есть не имеет корней. Например уравнение не имеет корней, поскольку при любом значении x , левая часть уравнения не будет равна правой части. Например, пусть . Тогда уравнение примет следующий вид

Пример 2 . Решить уравнение

Раскроем скобки в левой части равенства:

Приведем подобные слагаемые:

Видим, что левая часть не равна правой части. И так будет при любом значении y . Например, пусть y = 3 .

Буквенные уравнения

Уравнение может содержать не только числа с переменными, но и буквы.

Например, формула нахождения скорости является буквенным уравнением:

Данное уравнение описывает скорость движения тела при равноускоренном движении.

Полезным навыком является умение выразить любой компонент, входящий в буквенное уравнение. Например, чтобы из уравнения определить расстояние, нужно выразить переменную s .

Умножим обе части уравнения на t

В правой части переменные t сократим на t

В получившемся уравнении левую и правую часть поменяем местами:

У нас получилась формула нахождения расстояния, которую мы изучали ранее.

Попробуем из уравнения определить время. Для этого нужно выразить переменную t .

Умножим обе части уравнения на t

В правой части переменные t сократим на t и перепишем то, что у нас осталось:

В получившемся уравнении v × t = s обе части разделим на v

В левой части переменные v сократим на v и перепишем то, что у нас осталось:

У нас получилась формула определения времени, которую мы изучали ранее.

Предположим, что скорость поезда равна 50 км/ч

v = 50 км/ч

А расстояние равно 100 км

s = 100 км

Тогда буквенное примет следующий вид

Из этого уравнения можно найти время. Для этого нужно суметь выразить переменную t . Можно воспользоваться правилом нахождения неизвестного делителя, разделив делимое на частное и таким образом определить значение переменной t

либо можно воспользоваться тождественными преобразованиями. Сначала умножить обе части уравнения на t

Затем разделить обе части на 50

Пример 2 x

Вычтем из обеих частям уравнения a

Разделим обе части уравнения на b

a + bx = c , то у нас будет готовое решение. Достаточно будет подставить в него нужные значения. Те значения, которые будут подставляться вместо букв a, b, c принято называть параметрами . А уравнения вида a + bx = c называют уравнением с параметрами . В зависимости от параметров, корень будет меняться.

Решим уравнение 2 + 4x = 10 . Оно похоже на буквенное уравнение a + bx = c . Вместо того, чтобы выполнять тождественные преобразования, мы можем воспользоваться готовым решением. Сравним оба решения:

Видим, что второе решение намного проще и короче.

Для готового решения необходимо сделать небольшое замечание. Параметр b не должен быть равным нулю (b ≠ 0) , поскольку деление на ноль на допускается.

Пример 3 . Дано буквенное уравнение . Выразите из данного уравнения x

Раскроем скобки в обеих частях уравнения

Воспользуемся переносом слагаемых. Параметры, содержащие переменную x , сгруппируем в левой части уравнения, а параметры свободные от этой переменной — в правой.

В левой части вынесем за скобки множитель x

Разделим обе части на выражение a − b

В левой части числитель и знаменатель можно сократить на a − b . Так окончательно выразится переменная x

Теперь, если нам попадется уравнение вида a(x − c) = b(x + d) , то у нас будет готовое решение. Достаточно будет подставить в него нужные значения.

Допустим нам дано уравнение 4(x − 3) = 2(x + 4) . Оно похоже на уравнение a(x − c) = b(x + d) . Решим его двумя способами: при помощи тождественных преобразований и при помощи готового решения:

Для удобства вытащим из уравнения 4(x − 3) = 2(x + 4) значения параметров a , b , c , d . Это позволит нам не ошибиться при подстановке:

Как и в прошлом примере знаменатель здесь не должен быть равным нулю (a − b ≠ 0) . Если нам встретится уравнение вида a(x − c) = b(x + d) в котором параметры a и b будут одинаковыми, мы сможем не решая его сказать, что у данного уравнения корней нет, поскольку разность одинаковых чисел равна нулю.

Например, уравнение 2(x − 3) = 2(x + 4) является уравнением вида a(x − c) = b(x + d) . В уравнении 2(x − 3) = 2(x + 4) параметры a и b одинаковые. Если мы начнём его решать, то придем к тому, что левая часть не будет равна правой части:

Пример 4 . Дано буквенное уравнение . Выразите из данного уравнения x

Приведем левую часть уравнения к общему знаменателю:

Умножим обе части на a

В левой части x вынесем за скобки

Разделим обе части на выражение (1 − a )

Линейные уравнения с одним неизвестным

Рассмотренные в данном уроке уравнения называют линейными уравнениями первой степени с одним неизвестным .

Если уравнение дано в первой степени, не содержит деления на неизвестное, а также не содержит корней из неизвестного, то его можно назвать линейным. Мы еще не изучали степени и корни, поэтому чтобы не усложнять себе жизнь, слово «линейный» будем понимать как «простой».

Большинство уравнений, решенных в данном уроке, в конечном итоге сводились к простейшему уравнению, в котором нужно было произведение разделить на известный сомножитель. Таковым к примеру является уравнение 2(x + 3) = 16 . Давайте решим его.

Раскроем скобки в левой части уравнения, получим 2x + 6 = 16. Перенесем слагаемое 6 в правую часть, изменив знак. Тогда получим 2x = 16 − 6. Вычислим правую часть, получим 2x = 10. Чтобы найти x , разделим произведение 10 на известный сомножитель 2. Отсюда x = 5.

Уравнение 2(x + 3) = 16 является линейным. Оно свелось к уравнению 2x = 10 , для нахождения корня которого потребовалось разделить произведение на известный сомножитель. Такое простейшее уравнение называют линейным уравнением первой степени с одним неизвестным в каноническом виде . Слово «канонический» является синонимом слов «простейший» или «нормальный».

Линейное уравнение первой степени с одним неизвестным в каноническом виде называют уравнение вида ax = b.

Полученное нами уравнение 2x = 10 является линейным уравнением первой степени с одним неизвестным в каноническом виде. У этого уравнения первая степень, одно неизвестное, оно не содержит деления на неизвестное и не содержит корней из неизвестного, и представлено оно в каноническом виде, то есть в простейшем виде при котором легко можно определить значение x . Вместо параметров a и b в нашем уравнении содержатся числа 2 и 10. Но подобное уравнение может содержать и другие числа: положительные, отрицательные или равные нулю.

Если в линейном уравнении a = 0 и b = 0 , то уравнение имеет бесконечно много корней. Действительно, если a равно нулю и b равно нулю, то линейное уравнение ax = b примет вид 0x = 0 . При любом значении x левая часть будет равна правой части.

Если в линейном уравнении a = 0 и b ≠ 0 , то уравнение корней не имеет. Действительно, если a равно нулю и b равно какому-нибудь числу, не равному нулю, скажем числу 5, то уравнение ax = b примет вид 0x = 5 . Левая часть будет равна нулю, а правая часть пяти. А ноль не равен пяти.

Если в линейном уравнении a ≠ 0 , и b равно любому числу, то уравнение имеет один корень. Он определяется делением параметра b на параметр a

Действительно, если a равно какому-нибудь числу, не равному нулю, скажем числу 3 , и b равно какому-нибудь числу, скажем числу 6 , то уравнение примет вид .
Отсюда .

Существует и другая форма записи линейного уравнения первой степени с одним неизвестным. Выглядит она следующим образом: ax − b = 0 . Это то же самое уравнение, что и ax = b

Понравился урок?
Вступай в нашу новую группу Вконтакте и начни получать уведомления о новых уроках

Уравнение - это равенство, содержащее букву, значение которой надо найти.

В уравнениях неизвестное обычно обозначается строчной латинской буквой. Чаще всего используют буквы « x » [икс] и « y » [игрек].

  • Корень уравнения - это значение буквы, при котором из уравнения получается верное числовое равенство.
  • Решить уравнение - значит найти все его корни или убедиться, что корней нет.
  • Решив уравнение, всегда после ответа записываем проверку.

    Информация для родителей

    Уважаемые родители, обращаем ваше внимание на то, что в начальной школе и в 5 классе дети НЕ знают тему «Отрицательные числа».

    Поэтому они должны решать уравнения, используя только свойства сложения, вычитания, умножения и деления. Методы решения уравнений для 5 класса приведены ниже.

    Не пытайтесь объяснить решение уравнений через перенос чисел и букв из одной части уравнения в другую с изменением знака.

    Освежить знания по понятиям, связанным со сложением, вычитанием, умножением и делением вы можете в уроке «Законы арифметики».

    Решение уравнений на сложение и вычитание

    Как найти неизвестное
    слагаемое

    Как найти неизвестное
    уменьшаемое

    Как найти неизвестное
    вычитаемое

    Чтобы найти неизвестное слагаемое, надо от суммы отнять известное слагаемое.

    Чтобы найти неизвестное уменьшаемое, надо к разности прибавить вычитаемое.

    Чтобы найти неизвестное вычитаемое, надо от уменьшаемого отнять разность.

    x + 9 = 15
    x = 15 − 9
    x = 6
    Проверка

    x − 14 = 2
    x = 14 + 2
    x = 16
    Проверка

    16 − 2 = 14
    14 = 14

    5 − x = 3
    x = 5 − 3
    x = 2
    Проверка

    Решение уравнений на умножение и деление

    Как найти неизвестный
    множитель

    Как найти неизвестное
    делимое

    Как найти неизвестный
    делитель

    Чтобы найти неизвестный множитель, надо произведение разделить на известный множитель.

    Чтобы найти неизвестное делимое, надо частное умножить на делитель.

    Чтобы найти неизвестный делитель, надо делимое разделить на частное.

    y · 4 = 12
    y = 12: 4
    y = 3
    Проверка

    y: 7 = 2
    y = 2 · 7
    y = 14
    Проверка

    8: y = 4
    y = 8: 4
    y = 2
    Проверка

    Уравнение - это равенство, содержащее букву, знамение которой нужно найти. Решение уравнения - это тот набор значений букв, при котором уравнение превращается в верное равенство:

    Напомним, что для решения уравнении надо слагаемые с неизвестным перенести в одну часть равенства, а числовые слагаемые в другую, привести подобные и получить такое равенство:

    Из последнего равенства определим неизвестное по правилу: «один из множителей равен частному, деленному на второй множитель».

    Так как рациональные числа а и Ь могут иметь одинаковые и разные знаки, то знак неизвестного определяется по правилам деления рациональных чисел.

    Порядок решения линейных уравнений

    Линейное уравнение необходимо упростить, раскрыв скобки и выполнив действия второй ступени (умножение и деление).

    Перенести неизвестные в одну сторону от знака равенства, а числа - в другую сторону от знака равенства, получив тождественное заданному равенство,

    Привести подобные слева и справа от знака равенства, получив равенство вида ax = b .

    Вычислить корень уравнения (найти неизвестное х из равенства x = b : a ),

    Выполнить проверку, подставив неизвестное в заданное уравнение.

    Если получим тождество в числовом равенстве, то уравнение решено верно.

    Особые случаи решения уравнений

  1. Если уравнение задано произведением, равным 0, то для его решения используем свойство умножения: «произведение равно нулю, если один из сомножителей или оба сомножителя равны нулю».
  2. 27 (x - 3) = 0
    27 не равно 0, значит x - 3 = 0

    У второго примера два решения уравнения, так как
    это уравнение второй степени:

    Если коэффициенты уравнения являются обыкновенными дробями, то прежде всего надо избавиться от знаменателей. Для этого:

    Найти общий знаменатель;

    Определить дополнительные множители для каждого члена уравнения;

    Умножить числители дробей и целые числа на дополнительные множители и записать все члены уравнения без знаменателей (общий знаменатель можно отбросить);

    Перенести слагаемые с неизвестными в одну часть уравнения, а числовые слагаемые - в другую от знака равенства, получив равносильное равенство;

    Привести подобные члены;

    Основные свойства уравнений

    В любой части уравнения можно приводить подобные слагаемые или раскрывать скобку.

    Любой член уравнения можно переносить из одной части уравнения в другую, изменив его знак на противоположный.

    Обе части уравнения можно умножать (делить) на одно и то же число, кроме 0.

    В примере выше для решения уравнения были использованы все его свойства.

    Уравнения на умножение

    1) Формировать умение строить алгоритм на примере построения алгоритма решения простых уравнений на умножение, формировать умение использовать построенный алгоритм при решении уравнения.

    2) Тренировать вычислительный навык, решать текстовые задачи.

    Мыслительные операции, необходимые на этапе проектирования: анализ, синтез, сравнение, аналогия.

    1 этап. Мотивация к учебной деятельности

    1) мотивировать учащихся к учебной деятельности,

    2) определить содержательные рамки урока.

    Организация учебного процесса на этапе 1:

    — Какую тему мы сейчас изучаем на уроках математики? (Умножение и деление)

    — В каких заданиях применяем эти действия? (В решении примеров, задач)

    — Хотите узнать, какие еще есть задания, в которых мы можем использовать эти действия? (Да)

    Ребята, посмотрите, кто сегодня пришел к нам на урок? Вы их узнали? Что вы знаете об этих героях? (…)

    (Появляются знаки вопроса). Что происходит? Колобки озадачены и расстроены. Они хотели выполнить задание, а у них впервые не получилось. Они не знают, как открывать новые знания. Поможем? (…)

    А можно ли приниматься за работу с таким настроением, как у колобков? (Нельзя, не будет результата)

    Давайте улыбнемся друг другу и пожелаем удачи! Ну что же, будем действовать по плану открытия нового знания. Вам он хорошо знаком.

    2 этап. Актуализация знаний и фиксация затруднения в пробном действии

    1) актуализация изученных способов действий, достаточных для построения, их вербальная и знаковая фиксация и обобщение;

    2) актуализация мыслительных и познавательных процессов, достаточных для построения нового знания;

    3) мотивация к пробному учебному действию и его самостоятельному осуществлению;

    4) фиксация учащимися индивидуальных затруднений в выполнении пробного учебного действия или его обосновании.

    Организация учебного процесса на этапе 2:

    1) Актуализация формул нахождения площади и неизвестной стороны прямоугольника.

    С чего начнем? (С повторения). Мы должны повторить все, что знаем? (Нет, только то, что нам пригодится для открытия нового знания)

    — Что нужно найти в этом задании? (Площадь прямоугольника)

    — Как найти площадь прямоугольника? (Чтобы найти площадь прямоугольника, надо длину умножить на ширину)

    Появляется формула площади.

    Учащиеся выполняют задание.

    — Чему равна площадь? (18 кв. м)

    — Кто получил другой ответ?

    — В чем ваша ошибка?

    — Как найти неизвестную сторону прямоугольника? (Чтобы найти неизвестную сторону прямоугольника надо площадь разделить на известную сторону)

    — Появляется формула нахождения неизвестной стороны прямоугольника.

    — Составьте обратную задачу, в которой нужно найти длину прямоугольника (…)

    — Запишем решение обратной задачи.

    Ученик, составивший обратную задачу, решает ее на доске: 18:3=6(м) – длина

    — Теперь составьте другую обратную задачу.

    Ученик, составивший обратную задачу, решает ее на доске: 18:6=3 (м) – ширина

    У кого в этом задании не было ошибок? Поставьте себе знак + на маршрутном листе рядом с повторением. Кто допустил ошибку? Почему возникла ошибка? Вы поняли ее причину? Исправьте ошибку. Что вы себе поставите? (? и +).

    2) Актуализация алгоритма решения уравнений на сложение и вычитание.

    — Запишите: сумма Х + 5 равна 7. Как можно назвать эту запись? (Уравнение)

    — Что такое уравнение? (Равенство, в котором есть неизвестное число, называют уравнением)

    — Что поможет нам решить это уравнение? (Эталон решения уравнений на сложение)

    Один ученик у доски с комментированием. (Обозначу компоненты уравнения, подчеркну части, целое (сумму) обведу. Вижу, что неизвестна часть. Чтобы найти неизвестную часть, надо из суммы вычесть известную часть.

    У кого в этом задании не было ошибок? Поставьте себе знак + на маршрутном листе рядом с повторением. Кто допустил ошибку? Почему возникла ошибка? Вы поняли ее причину? Исправьте ошибку. Что вы себе поставите? (- и +).

    — Почему мы повторили именно это? (Это пригодится нам для открытия нового знания)

    — Какой следующий шаг? (Пробное действие) Для чего оно нужно? (Чтобы понять, чего мы не знаем)

    Учитель раздает учащимся карточки с заданием для пробного действия:

    — Какое задание нужно выполнить? (Решить уравнение)

    — С каким действием? (С умножением)

    — А что нового в этом задании? (Мы не решали уравнения на умножение)

    Попробуйте выполнить это задание. (30 сек.)

    — Кто не выполнил задание?

    Что вы не смогли сделать? (Мы не смогли решить уравнение)

    — Кто нашел корень уравнения? Какие результаты у вас получились?

    Учитель фиксирует результаты на доске рядом с пробным действием

    — Обоснуйте свое мнение.

    Что вы не можете сделать? (Мы не можем обосновать свой ответ.)

    У вас возникло. (затруднение). Поставим… (знак вопроса) рядом с пробным действием на маршрутном листе.

    — Какой следующий шаг на уроке? (Разобраться, в чем у нас затруднение)

    — А раз возникло затруднение, надо…(Остановиться и подумать)

    3 этап. Выявление места и причины затруднения

    1) восстановить выполненные операции и зафиксировать место затруднения;

    2) соотнести свои действия с используемым способом действий и на этой основе выявить и зафиксировать во внешней речи причину затруднения.

    Организация учебного процесса на этапе 3:

    — Какое задание вы должны были выполнить? (Мы должны были решить уравнение на умножение)

    — Как рассуждали, выполняя пробное действие? (Пытались воспользоваться известным алгоритмом решения уравнений …)

    — В чем затруднение? (Алгоритм не подходит)

    Почему же возникло затруднение? (У нас нет способа для решения уравнений на умножение)

    Вы поняли, чего вы не знаете? (Да). Поставьте себе знак + на маршрутном листе рядом с третьим шагом.

    4 этап. Построение проекта выхода из затруднения

    1) согласовать и зафиксировать цель и тему урока;

    2) построить план и определить средства достижения цели.

    Организация учебного процесса на этапе 4:

    — Мы поняли, чего мы не знаем, теперь можем… (Сами открывать способ)

    Сначала нужно поставить цель. Если вы не знаете способа решения уравнений на умножение, значит, ваша цель… (Открыть способ решения таких уравнений)

    — Сформулируйте тему нашего урока (…)

    Написать тему на доске:

    — Будем действовать, как настоящие сыщики. Составим план действий. Слайд

    — Давайте подумаем, что нам может помочь. Вспомните, вы повторили в самом начале урока. (Алгоритм решения уравнений на сложение, формулу нахождения площади)

    — Какая формула может нам помочь? (Формула нахождения площади и неизвестной стороны прямоугольника)

    — Пробуем применить формулу площади прямоугольника.

    — Предлагаю воспользоваться известным вам алгоритмом решения уравнений на сложение.

    Алгоритм.

  3. Выделяю целое и части.
  4. Что неизвестно?
  5. Применяю правило.
  6. Нахожу неизвестное х.
  7. Что в этом алгоритме вам явно не подходит? (1 пункт)
  8. Когда у вас были уравнения на сложение, вы их компоненты соотносили с частями и целым, используя отрезки. А с чем вы соотносили компоненты умножения? (С площадью)
  9. Что будете использовать вместо отрезка? (Моделью прямоугольника)

Заменим п.1 на Обозначим компоненты уравнения на модели прямоугольника.

— Остальные пункты алгоритма вам подходят?

— Используя этот алгоритм, можно попробовать решить уравнение?

— Что сделаем, чтобы было удобно пользоваться этим правилом всегда? (Запишем правило в общем виде)

Запишем правило в общем виде.

— Какими средствами будем пользоваться?

Пробуем применить формулу площади прямоугольника…

Средства: модель прямоугольника, алгоритм.

5 этап. Реализация построенного проекта

1) реализовать построенный проект в соответствии с планом;

2) зафиксировать способы записи выражений на эталоне;

3) организовать фиксацию преодоления затруднения;

4) организовать уточнение общего характера нового знания.

Организация учебного процесса на этапе 5:

Я предлагаю поработать вам в группах. Назовите правила работы в группах.

Правила работы в группах

1. В группе должен быть ответственный.

2. Один говорит, другие слушают.

3. Свое несогласие высказывать вежливо..

4. Работать должны все.

Учащиеся объединяются в группы.

— Выполните план в группах.

Ответственный от каждой группы получает задание.

1. Воспользуюсь моделью прямоугольника, нанесу компоненты уравнения на модель.

2. Применю правило площади прямоугольника. (Чтобы найти неизвестную сторону прямоугольника надо площадь разделить на известную сторону)

3. Найду корень уравнения

Мы обозначили на модели прямоугольника числа. Видно, что неизвестна сторона прямоугольника. Чтобы найти неизвестную сторону прямоугольника, надо площадь разделить на известную сторону. Выполнили вычисления и нашли корень уравнения, х=5.

— Что осталось сделать по плану? (Записать уравнение в общем виде)

— Как записать уравнение в общем виде? (С помощью букв латинского алфавита)

— Как обозначите в уравнении числа, которые являются сторонами прямоугольника? (Подчеркнем)

— Число, которое является площадью, предлагаю взять в прямоугольник, почему это удобно? (Напоминает о формуле, которой мы пользуемся)

— Нужно ли будет составлять другой эталон для случая, где х стоит на месте другого множителя? (Нет)

— Почему? (Можно воспользоваться переместительным свойством умножения)

— Как проверить свое открытие? Какие ключи к знаниям у нас есть? (Посмотреть в учебнике)

Откройте учебники на стр.1. Прочитайте правило.

Молодцы! Вы помогли колобкам. Слайд (аплодисменты).

Давайте теперь вернемся к пробному действию.

Дописать необходимое на доске.

Смогли вы преодолеть затруднение? (Да). Поставим себе знак + на маршрутном листе.

На обычной доске под шагом “Сам найду способ” прикрепить новые эталоны.

Что вы теперь сможете делать с помощью новых знаний? (Решать уравнения)

6 этап. Первичное закрепление

1) организовать усвоение детьми нового способа действий при решении уравнений на умножение с их проговариванием во внешней речи.

Организация учебного процесса на этапе 6:

1) Фронтальная работа. На доске левая часть-алгоритм, правая – уравнение+модель.

2) 4 · х=8; 3 · х=9; х · 4=12.

3) Учитель открывает на доске задание на закрепление. Учащиеся по цепочке выходят к доске и выполняют задание с комментированием. Вариант комментирования:

— Сначала обозначу площадь прямоугольника квадратом, а стороны подчеркну. В данном уравнении неизвестна сторона прямоугольника. Значит, надо площадь прямоугольника разделить на известную сторону. Восемь разделить на 4 будет 2, х равен 2.

Дальнейшее выполнение задания комментируется аналогично.

Физминутка гимнастика для глаз.

Мы немного отдохнём. и на всё ответ найдём.
На носочки встанем, руки вверх потянем.
Руки на пояс, наклоны вперёд.
Теперь попрыгаем, и сядем на места!

Сейчас все отдохнули, и новая забота:

Нужно сделать на “отлично” парную работу.

Учитель раздает карточки с заданием для работы в парах.

Учащиеся выполняют задания в парах с комментированием. Проверка организуется по образцу Д-7.

— Проверьте свои результаты.

Исправьте ошибки. У кого в этом задании не было ошибок? Поставьте себе знак + на маршрутном листе рядом с 5-м шагом. Кто допустил ошибку? Почему возникла ошибка? Вы поняли ее причину? Исправьте ошибку. Что вы себе поставите? (? и +)

— Какой следующий шаг на уроке? (Проверить себя, справимся ли мы самостоятельно)

7 этап. Самоконтроль с самопроверкой по эталону

1) тренировать способность к самоконтролю и самооценке;

2) проверить умение решать уравнения на умножение.

Организация учебного процесса на этапе 7:

— Выполните данные уравнения самостоятельно. Учащиеся выполняют самостоятельную работу на карточках

— Проверка организуется по эталону Д-8.

— Сделайте вывод. (Нужно еще потренироваться.)

— Сделайте вывод. (Мы все хорошо усвоили.)

— У кого в этом задании не было ошибок? Поставьте себе знак + на маршрутном листе рядом с 5-м шагом. Кто допустил ошибку? Почему возникла ошибка? Вы поняли ее причину? Исправьте ошибку. Что вы себе поставите? (? и +).

8 этап. Включение в систему знаний и повторение

1) включить новое знание в систему знаний;

2) тренировать умение решать задачи.

Организация учебного процесса на этапе 8:

— Что нужно знать, чтобы правильно решать уравнения на умножение? (Таблицу умножения и деления, формулу площади). Предлагаю вам решить задачу №4 стр.2.

Учащиеся выполняют задание. Проверка организуется по образцу Д-9.

— Кто из вас ошибся?

— В чем ошибка? (В выборе правила, в вычислениях, …)

9 этап. Рефлексия учебной деятельности на уроке

Цели:

1) зафиксировать новое содержание, изученное на уроке;

2) оценить свою работу и работу класса на уроке;

4) наметить направления будущей учебной деятельности;

3) обсудить домашнее задание.

Организация учебного процесса на этапе 9:

— Какую цель вы перед собой ставили? (…)

— Достигли ли вы цели? (Докажите)

— Я предлагаю вам оценить свою работу на уроке. Посмотрите еще раз на свои планы урока, посмотрите, сколько у вас плюсов.

— На обычной доске изображение колобков по отдельности. Один улыбается. Те из вас, кто считает, что понял и запомнил новую тему, возьмите восклицательные знаки и прикрепите их рядом с улыбающимся Колобком. Те, кто в чем-то еще не уверен, у кого остались вопросы, кто допустил ошибки в самостоятельной работе – прикрепите вопросительный знак рядом с серьезным Колобком. Вы потренируетесь и обязательно преодолеете свое затруднение.

— Вы сегодня очень хорошо поработали, но значит ли это, что больше не надо тренироваться? (Надо выполнить домашнюю работу)

xn--i1abbnckbmcl9fb.xn--p1ai

Решение уравнений умножением

Неизвестная величина может быть связана с известной величиной не только знаком + или -, но может быть разделена на какую-нибудь величину, как в этом уравнении: $\frac = b$.

Здесь решение не может быть найдено, как в предыдущих примерах, переносом члена уравнения. Но если оба члена уравнения умножить на a, уравнение примет вид
$x = ab.$

То есть, знаменатель дроби в левой части сокращается. Это может быть доказано свойствами дробей.

Когда неизвестная величина разделена на известную величину, уравнение решается путем умножения каждой стороны на эту известную величину.

Те же самые переносы должны быть сделаны в этом случае, как и в предыдущих примерах. Однако надо помнить, что умножать необходимо каждый член уравнения.

Пример 1. Решите уравнение $\frac + a = b + d$
Умножаем обе стороны на $c$
Произведение будет $x + ac = bc + cd$
И $x = bc + cd — ac$.

Пример 1. Решите уравнение $\frac + d = h$
Умножаем на $a + b$ $x + ad + bd = ah + bh$.
И $x = ag + bh — ad — bd.$

Когда неизвестное значение находится в знаменателе дроби, уравнение решается похожим способом, то есть умножением уравнения на знаменатель.

Пример 3. Решите уравнение $\frac + 7 = 8$
Умножая на $10 — x$ $6 + 70 — 7x = 80 — 8x$
Тогда $x = 4$.

Хотя это и не обязательно , но часто очень удобно избавиться от знаменателя дроби, состоящего только из известных величин. Это можно сделать, похожим способом, когда избавляются от знаменателя, включающего в себя неизвестную величину.

Возьмем для примера $\frac = \frac + \frac $
Умножаем на a $x = \frac + \frac $
Умножаем на b $bx = ad + \frac $
Умножаем на c $bcx = acd + abh$.

Или, мы можем умножить на произведение всех знаменателей сразу.

В этом же самом уравнении $\frac = \frac + \frac $
Умножаем члены на abc $\frac = \frac + \frac $

После сокращения каждого одинакового значения в одной дроби, получим $bcx = acd + abh$, как и в предыдущем варианте. Отсюда,

В уравнении можно избавиться от дробей , умножая каждую сторону уравнения на все знаменатели .

При избавлении от дробей в уравнении необходимо соблюдать правильность написания знаков и коэффициентов каждой дроби в процессе раскрытия скобок

Карточка-шпаргалка «Решение уравнений. Как найти неизвестное», умножение и деление, 11х20 см


  • Характеристики
  • Описание
  • Задать вопрос
  • Оставить отзыв
    • Общие
    • Торговая марка Атмосфера праздника
    • Артикул 1060173
    • Сертификат Не подлежит сертификации
    • Страна Россия
    • Упаковка и фасовка
    • В боксе 2000 шт
    • Фасовка по 20 шт
    • Индивидуальная упаковка Без упаковки
    • Размер упаковки 0,1 см × 6 см × 13 см
    • Габариты и вес
    • Размер 0,1 см × 7 см × 13 см
    • Вес 3 г
    • Особенности
    • Плотность, г/м² 190
    • Отделка Без отделки
    • Для кого Унисекс
    • Тематика праздника Без повода
    • Адресат Без адресата
    • Материал Картон
    • Школьный предмет Математика
    • Россия входит в десятку самых читающих стран мира! Интерес к чтению у наших соотечественников растёт из года в год, что не может не радовать, ведь это прекрасная и очень полезная привычка.

      Изучая различную литературу, вы можете получить очень много ценной информации, расширить кругозор, словарный запас и стать эрудированным. Кроме того, книга - это отличный способ расслабиться и с удовольствием провести время. Пусть Карточка-шпаргалка «Решение уравнений. Как найти неизвестное», умножение и деление, 11х20 см станет очередным полезным изданием в вашей коллекции.

      Сима-ленд вправе самостоятельно и без уведомления пользователей отбирать вопросы для публикации. Мы не размещаем вопросы, которые:

    • не относятся к тематике работы магазина, осуществлению покупок в нём;
    • содержат ненормативную лексику, высказывания оскорбительного характера;
    • Мы не публикуем вопросы, в которых содержатся:

    • ссылки на другие веб-сайты, а также упоминания конкретных продавцов и импортёров товаров;
    • Сима-ленд оставляет за собой право удалить опубликованный вопрос в любое время, а также самостоятельно определять срок, в течение которого вопросы считаются актуальными и на который они публикуются в рамках сайта Сима-ленд.

      Мы не принимаем на себя обязательств сообщать пользователям о причинах отклонения вопросов и удаления ранее опубликованных вопросов.

      Если пользователь задаёт вопрос, он соглашается получать уведомления от сайта Сима-ленд о новых ответах на свои вопросы.

      Сима-ленд вправе самостоятельно и без уведомления пользователей отбирать отзывы для публикации. Мы не размещаем отзывы, которые:

    • не относятся к реальному опыту использования данного товара;
    • не содержат полезной информации для других пользователей;
    • содержат ссылки на другие веб-сайты.
    • Мы не публикуем подборки и обзоры товаров, в которых содержатся:

    • ссылки на другие веб-сайты в тексте подборки и обзора, а также упоминания конкретных продавцов и импортёров товаров;
    • утверждения, порочащие честь, достоинство и деловую репутацию третьих лиц (в том числе магазинов, производителей и импортёров товаров);
    • материалы (в том числе в виде текста, видео, графических изображений, кода), нарушающие права третьих лиц, в том числе права на результаты интеллектуальной деятельности и средства индивидуализации.
    • Сима-ленд оставляет за собой право удалить опубликованный отзыв, подборку и обзор товаров в любое время, а также самостоятельно определять срок, в течение которого отзывы считаются актуальными и на который они публикуются в рамках сайта Сима-ленд.

      Мы не принимаем на себя обязательств сообщать пользователям о причинах отклонения публикации и удаления ранее опубликованных отзывов, оценок, подборок и обзоров товаров.

      Если пользователь отвечает на отзыв или вопрос к нему, он соглашается получать уведомления от сайта Сима-ленд о новых ответах на свои комментарии.

      www.sima-land.ru

      • Программа летнего оздоровительного лагеря с дневным пребыванием детей Составители: Пилипей О.Н. (1 кв.категория) Мелентьева И.Н. (1 кв. категория) Демидова О.Б. (1 кв. категория) Возраст детей: 5 -15 лет Срок […]
      • Как в налоговом учете отразить продажу основных средств При продаже основных средств оформите первичные учетные документы, утвержденные постановлением Госкомстата России от 21 января 2003 г. № 7 (ст. 2, 5, […]
      • Налог на проценты по вкладам: придется платить? Налоги на проценты по вкладам физических лиц в России действуют и сегодня. В каких случаях клиент должен заплатить налоги с процентных доходов по депозитам? С […]